Fundamentals of Treatment:

Understanding Obesity

5 Objectives

• Recognize why obesity is a disease • Understand the different definitions of obesity • Learn the basic regulators of hunger and satiety • Identify adiposity-related diseases • Facilitate appropriate discussion of obesity with patients • Recognize the essential history and physical exam components of an obesity evaluation • Learn the essential diagnostic tests used in evaluation the patient with obesity

11

Prevalence¶ of Self-Reported Obesity Among U.S. Adults by State and Territory, BRFSS, 2017

¶ Prevalence estimates reflect BRFSS methodological changes started in 2011. These estimates should not be compared to prevalence estimates before 2011.

*Sample size <50 or the relative standard error (dividing the standard error by the prevalence) ≥ 30%.

12 Centers for Disease Control and Prevention. 2017 Obesity Prevalence Maps Prevalence of Self-Reported Obesity Among Non-Hispanic White Adults, by State and Territory, BRFSS, 2015-2017

*Sample size <50 or the relative standard error (dividing the standard error by the prevalence) ≥ 30%.

13 Centers for Disease Control and Prevention. 2017 Obesity Prevalence Maps

Prevalence of Self-Reported Obesity Among Hispanic Adults, by State and Territory, BRFSS, 2015-2017

*Sample size <50 or the relative standard error (dividing the standard error by the prevalence) ≥ 30%.

14 Centers for Disease Control and Prevention. 2017 Obesity Prevalence Maps Prevalence of Self-Reported Obesity Among Non-Hispanic Black Adults, by State and Territory, BRFSS, 2015-2017

*Sample size <50 or the relative standard error (dividing the standard error by the prevalence) ≥ 30%.

15 Centers for Disease Control and Prevention. 2017 Obesity Prevalence Maps

Obesity bias among health care professionals

• In a survey of US primary care , genetic factors ranked below physical inactivity, overeating • A survey of US-based cardiologists, endocrinologists, and primary care providers showed that about half agreed that obesity is a due to a lack of self-control • More than 30% viewed patients with obesity as weak-willed, sloppy, or lazy • Over 50% viewed patients with obesity as awkward, unattractive, ugly, and noncompliant • Medical students reported that patients with obesity are a common target of negative attitudes and derogatory humor by peers (63%), health-care providers (65%), and instructors (40%).

Hollman BMC (2019) 19:16 16 Puhl, et al Obesity (Silver Spring). 2014 Consequences of obesity bias

Weight stigma has been shown to increase eating, decrease self-regulation, increase resistance to exercise, and raise cortisol levels.

Provider negative attitudes toward patients about obesity may result in:

• Avoidance of medical care • Mistrust of the medical care provider • Decreased adherence to recommendations • Poor communication in patient/provider communication

“Believing that obesity is caused by factors outside a person’s control was positively correlated with proficiency in obesity counseling skills.”

Phalen, SM Obes Rev. 2015 Apr; 16(4): 319–326 Fang, V et al BMC Obes. 2019; 6: 5 17 Tomayama, AJ et al BMC Med. 2018; 16: 123

Definition of Obesity

Obesity is defined as a chronic, relapsing, multi- factorial, neurobehavioral disease, wherein an increase in body fat promotes adipose tissue dysfunction and abnormal fat mass physical forces, resulting in adverse metabolic, biomechanical, and psychosocial health consequences.

18 Adapted from Obesity Algorithm®, Obesity Association® Obesity is a Multi-Factorial Disease

Neurobehavioral

Medical Genetics / Epigenetics

Endocrine Environment

Immune

19 Adapted from Obesity Algorithm®, Obesity Medicine Association®

Severity of Obesity – as defined by Body Mass Index

Body Mass Index (BMI)  Body weight in kilograms (Height in meters)²

Normal weight (18.5 – 24.9) and Obesity Overweight (25.0 – 29.9) Classification: Body Mass Index Class I obesity (30.0 – 34.9) Class II obesity (35.0 – 39.9) (BMI) in kg/𝑚* Class III obesity (≥ 40)

Adapted from Obesity Algorithm®, Obesity Medicine Association® * Different BMI cut-off points may be more appropriate for women versus men, among those of different races, and among individuals

20 Defining Patients with Obesity

Percent Category Female Male Body Essential 10-13% 2-5% Fat Athletic 14-20% 6-13% Fitness 21-24% 14-17% Acceptable 25-31% 18-24% Obese >31% >25%

21

Defining Patients with Obesity

• Waist Circumference

Race Males Females

Caucasian 40” 35”

African 37” 31.5” American

Latino 35.5” 31.5”

Asian 33.5” 31.5”

22 Adults with Overweight and/or Obesity

Is there clinical evidence that the increase in body fat is pathogenic?

Yes No

Sick fat disease or Fat mass Metabolically healthy (adiposopathy) disease but with obesity

Adapted from Obesity Algorithm®, Obesity Medicine Association® 23

Pathogenic Adiposity

Deranged endocrine and Abnormal and pathologic immune responses physical forces

Sick fat disease (SFD) Fat mass disease (FMD) (Adiposopathy)

Endocrine/Metabolic Biomechanical/Structural

24 Adapted from Obesity Algorithm®, Obesity Medicine Association® 25

26 Derived from Spadano et al, JAMA 1999 Pediatric Weight Assessment

Age <2 Age 2-20 BMI BMI Percentile

WHO or <5% CDC 5-84% 85-94% Growth 95-99% or Charts BMI>30 >120% of 99th% 27

Adiposopathy “Sick Fat Disease”

28 Metabolic Syndrome (MetS)

Metabolic Syndrome

Body size Hyperinsulinemia Waist Circumference Insulin Resistance Central adiposity

Dyslipidemia: Glucose Uric Acid Hemodynamic: Metabolism: TG Inflammation: Metabolism: SNS activity Uric acid PP lipidemia CRP IFG Na retention Urinary uric acid HDL-C Cytokines IGT Hypertension clearance Small dense LDL Fibrinogen

Information from Reaven G. Syndrome X: 10 years after. Drugs. 1999;58(suppl 1):19-20. Coronary Heart Disease

29

Unified Definition of Metabolic Syndrome (MetS) MetS identified by three or more of the following criteria: • Abdominal circumference – >40 inches men, >35 inches women (Caucasian data) – Population and country specific • Triglycerides ≥150mg/dL* • HDL low* – <40mg/dL men, <50mg/dL women • Elevated BP – systolic ≥130 and/or diastolic ≥ 85mmHg* (or currently prescribed an anti-hypertensive medication) • Glycemia FBS ≥100mg/dL* *Or on medication

Patients may have insulin resistance and not meet the diagnostic criteria for metabolic syndrome

(Alberti 2009) 30 Metabolic Syndrome and Risk of CVD

• Cardiovascular Disease – 2x increase risk of CV event – 1.5x increase risk of all-cause mortality • Marker for CV disease in addition to LDL – CV risk assessment is primarily focused on LDL – MetS reflects a pro-inflammatory state that also increases risk for CV disease

31

Age-adjusted Prevalence of Obesity and Diagnosed Diabetes Among US Adults

Obesity (BMI ≥30 kg/m2) 1994 2000 2015

No Data <14.0% 14.0%–17.9% 18.0%–21.9% 22.0%–25.9% > 26.0%

Diabetes 1994 2000 2015

CDC’s Division of Diabetes Translation. United States Surveillance System available at http://www.cdc.gov/diabetes/data 32 Prediabetes & Type 2 Diabetes Mellitus • T2DM is associated with obesity • Obesity causes insulin resistance • To keep glucose normal, the physiologic response to insulin resistance is hyperinsulinemia • Hyperinsulinemia contributes to weight gain • Hyperglycemia occurs due to β-cell failure and relative insulin deficiency • Up to 80% β-cell failure at diagnosis of T2DM

(DeFronzo 2009)

33

Prediabetes & Type 2 Diabetes Mellitus

↑ Endogenous Obesity insulin

Insulin resistance

34 Developing Diabetes: The Clinical Progression Insulin resistance begins 10 -15 years prior to dx of T2DM

Information from Ramlo-Halsted B. Clinical Diabetes. Spring 2000. 35

Diabetes Prevention Data

Incidence T2DM Lifestyle ↓ 58% Metformin ↓ 31%

Weight Loss 1 kg ↓ risk 16% 10% WL ↓ risk 85% at 3 years

DPP Research Group 2002, Hoskin, 2014 36 DPP Research Group 2015 Diabetes Prevention Program Outcome Studies (DPPOS)

• Showed continued benefit at 15 years • Early detection and lifestyle intervention is key • Continued weight gain increases risk of T2DM

DPP Research Group 2002, Hoskin, 2014 DPP Research Group 2015

37

Managing T2DM

• Weight loss reduces insulin resistance and improves glycemic control • Dietary changes and weight loss may require changes in diabetes • Diets with severe restriction in calories or carbohydrates often require pro-active changes in diabetes therapy

38 Biology of Appetite Regulation

39

Purposeful behavior regulates weight versus Biology regulates weight

40 Traditional Thinking Additional Understanding

Purposeful behavior Biology regulates weight Regulates weight - (+) - Calories in & Hormonal Response Calories out regulates weight regulates weight

41

CME Polling Questions

Instructions:

Go to www.pollEv.com/OMA248

Enter your FULL FIRST and LAST NAME for your username

The polling question will appear shortly

Participation in the polling questions is a requirement to earn ABIM MOC points for this course.

42 What’s the average annual calorie consumption for adult males?

• 150,000 kcal • 250,000 kcal • 400,000 kcal • 500,000 kcal • 750,000 kcal • 1,000,000 kcal

43

What is the average annual weight gain for adults?

•0.5 lb. •1.0 lb. •2.0 lbs. •3.0 lbs. •5.0 lbs.

46 Daily excess calories intake needed to gain 1 lb./year?

• 10 kcal • 20 kcal • 30 kcal • 40 kcal • 50 kcal

49

Weight Gain • Imbalance of energy intake and energy output – Estimated annual caloric requirement in middle-aged adults • Males ~______kcal/year • Females ~______kcal/year – Average annual weight gain for middle-aged adults • ___ lb(s)/year (variance of 0.35 to 0.93%) – Small changes in caloric intake greatly impact weight – Biologic caloric regulation is more likely to explain small annual variations in weight than purposeful control

Williamson 1993 52 https://www.cnpp.usda.gov/sites/default/files/usda_food_patterns/EstimatedCalorieNeedsPerDayTable.pdf Regulation of Hunger and Appetite

•Brain

•Gut

• Adipose Tissue

• Pancreas

53

Brain Regulation of Hunger and Satiety

Leptin Ghrelin Insulin PYY CCK Orexin Amlyn GLP-1 Adiponectin Vagus nerve plus more…

54 Gut Regulation of Hunger and Satiety • GI Hormones – Stomach • Ghrelin – increases hunger – Small intestine • CCK • GLP-1 All ↓ hunger and ↑ satiety • PYY } – Pancreas • Insulin - ↓ hunger and ↑ satiety • Amylin - ↓ hunger and ↑ satiety

55

Adipose Regulation of Hunger and Satiety

• Adipose Tissue – – ↓ hunger and ↑ satiety • Leptin levels drop quickly with calorie restriction

56 Hormonal adaptions to severe energy restriction and weight loss persist for at least one year

Sumithran P et al. N Engl J Med 2011;365:1597-1604 57

Adaptive hormonal changes result in long term enhanced perceptions of hunger and desire to eat

58 Sumithran P et al. N Engl J Med 2011;365:1597-1604 Adipose Regulation of Hunger and Satiety

• Take Home Message – Negative energy balance triggers a physiologic increase in hunger and decrease in satiety

EIN ↑ Hunger E OUT ↓ Satiety

59

Obesity is Multifactorial Disease

• Other influences on appetite – Hedonic response – – Emotional response – Stress response – Environmental stimuli – Genetic/epigenetic expression – Immune response

60 Starting the Discussion

• Approach obesity as a chronic disease

• Requires a thorough assessment and evaluation, treatment, and long-term treatment approach

• Simple changes in your communication techniques builds support and empathy

• Focus on the medical consequences of adiposity

Bocquier A. Obes Res. 2005. 61

5 A’s of Obesity Management

Arrange/ Ask Assess Advise Agree Assist • Ask for • Assess BMI, • Advise the • Agree on • Assist in permission to waist patient about realistic identifying and discuss body circumference, the health weight-loss addressing weight and obesity risks of expectations, barriers stage obesity, the targets, • Provide benefits of resources • Explore modest weight behavioral changes, • Assist in finding readiness for • Explore drivers loss (5-10%), and consulting change and the need for and specific details of the with appropriate complications long-term providers of excess strategy, and treatment treatment plan • Arrange a weight regular follow- options. up

Adapted from Obesity Algorithm®, Obesity Medicine Association®

62 Avoid Derogatory Language

Use Instead of Overweight Fat Unhealthy weight Obese Heavy Morbidly obese Eating habits / Nutrition Diet Physical activity Exercise

63

Assessment and Evaluation

History

64 History of Present Illness (HPI): Weight History

• Age of onset: Child, teen, adult, perimenopause?

• Weight range: Highest, lowest, or “best”?

• Rate of weight gain: Past month, year, decade?

• Life event triggers: Puberty, marriage, , divorce, occupation change?

• Current motivation

65

HPI

• Past Weight-loss Attempts? • Self-directed • “Popular” diets/programs, • Physical activity • Over-the-counter medications • Medically supervised programs • Anti-obesity medications • Meal replacement programs • What worked best? What didn’t work?

66 HPI - Nutrition

Eating Patterns/Problems • Mealtimes, snacks and beverages • Triggers: people, places, activities, emotions (stress, boredom, anger) • Fast food frequency • Reasons for eating out: convenience, work, leisure, enjoyment • Access to healthy food • Meal planning

67

HPI – Physical Activity • Current level of physical activity? • Favorite past activities? • Are there any barriers to physical activity?

68 Past Medical History: Weight-related Conditions

• Review adiposity-related diseases and secondary causes of obesity

• Hypertension, , metabolic syndrome, gout

• Type 2 diabetes, gestational DM, LGA babies, pre-diabetes and insulin resistance

• Fatty liver, polycystic ovary syndrome, fibromyalgia,

• Hypothyroidism

• Psychiatric: depression, anxiety

69

Past Medical History: Weight-related Conditions • Past surgical history • S/p bariatric (weight loss) • S/p cholecystectomy or other digestive system surgery • • Current medications • Monitoring: diabetes, hypertension, anti-coagulant meds • Weight positive: steroids, insulin, antidepressants • Some medications may preclude use of certain anti-obesity medications

70 PMH – Eating Disorders • Prior diagnosis of ? – Bulimia – disorder (BED) – nervosa – Night eating syndrome (NES) • Some have an eating disorder but do not meet the criteria • Some are unaware they have an eating disorder

71

Family History

• Obesity

• Type 2 diabetes, pre-diabetes

• Hypertension / hyperlipidemia / cardiovascular disease

• Cancer

• Thyroid disease / autoimmune disorders / other hormonal disorders

• Psychiatric - dysthymia, depression, alcoholism

72 Social History • Tobacco, alcohol, cannabis, other drug use • Occupation, work environment, work hours, schedule/night shift, commuting time and travel • Sleeping behavior • Birth control / desire to become pregnant • Social support

73

Obesity Related Review of Systems

• General: lack of energy, daytime sleepiness • HEENT: • Cardiac: chest pain or tightness, palpitations • Pulmonary: dyspnea on exertion, sleep apnea • GI: abdominal pain, reflux, bowel irregularity • GU: erectile dysfunction, ↓ libido, irregular menses, incontinence

74 Obesity Related Review of Systems

• Extremities: joint pains, leg swelling • Endo: polyuria, polydipsia, fatigue, cold intolerance • Neurologic: headaches, neuropathy, history of seizures • Skin: hirsutism, acne, rash • Mental health: depression, anxiety

75

Assessment and Evaluation

Physical Examination

76 Physical Examination

• Vital signs • Height • Weight: scales accurate to 600 pounds • Body Mass Index calculation (weight/height2) • Blood pressure – using appropriate cuff size for arm circumference • Pulse, respiratory rate

• Anthropometric measurements • Neck circumference • Waist at superior iliac crest, horizontal to floor

77

Physical Exam - Significant Findings

• HEENT • Facial plethora (red, puffy) - Cushing's • Hirsutism – Cushing's, PCOS • Periorbital , lateral thinning of eyebrows, conjunctival injection (red eyes), scalloped tongue – hypothyroidism •Oral • Pharynx obstruction – (OSA), Mallampati score • Mucosal abnormalities – vitamin deficiencies • Erosion of dental enamel, swollen parotid glands – bulimia nervosa •Neck • Enlarged thyroid or thyroid masses/nodules • Posterior cervical fat pad (buffalo hump) – Cushing’s • Carotid bruit • Neck circumference: ≥ 17” for males, ≥ 16” for females  increased risk for OSA

78 Physical Exam - Significant Findings

• Lungs • Decreased capacity, tidal volume • Wheezing - asthma • Heart • Rhythm abnormalities – atrial fibrillation • Murmurs • Abdomen • Tenderness or enlargement, liver size, ascites • Hernias/prior surgical scars • Red or violaceous striae - Cushing's versus obesity • Neurologic • Deep tendon reflexes • Delayed relaxation – hypothyroidism • Hyperactive - low magnesium

79

Physical Exam - Significant Findings

•Extremities • Edema - CHF, renal, liver disease, DVT, venous stasis • • Lipedema – “stove-pipe” legs, “nodular fat” • Painful fat – Dercum’s disease (adiposis dolorosa) • Muscle weakness/wasting - Cushing's • Dry cracking heels - DM, hypothyroid, essential fatty acid deficiency

80 Lymphedema and Lipedema

81

Physical Exam - Significant Findings

•Skin • Acanthosis nigricans (dark skin in the flexures of the neck, axilla, or groin) - insulin resistance/diabetes • Acne - PCOS, Cushing's • Acrochordons (skin tags) - insulin resistance/diabetes • Xanthomas or xanthelasmas – hyperlipidemia • Bruising - Cushing's, hypothyroidism, vitamin deficiencies • Hyperpigmentation - Cushing's • Dry, scaly - hypothyroidism, nutrient deficiency

82 Acanthosis Nigricans

Back of the Neck Axilla

83

Assessment and Evaluation

Testing

84 Tests: Looking for Causes and Co-morbidities • Complete blood cell count – R/O anemia, complications of bariatric surgery, etc • Comprehensive metabolic panel (electrolytes, renal and liver function tests, glucose) – important to have baseline values • Thyroid function tests - hypothyroidism • Fasting lipid profile – dyslipidemia or metabolic syndrome may be present • Urinalysis • Uric acid – underlying hyperuricemia may be present (hyperinsulinemia and diuretics increase risk) • Consider further testing for diabetes: hemoglobin A1c, glucose tolerance test, serum insulin, c-peptide, urine for microalbumin (esp DM and severe obesity) • Consider inflammatory markers: high sensitivity C-reactive protein • Consider measuring vitamin D or other vitamin levels if history of

85 gastric bypass surgery (See ASMBS recommendations in appendix)

Other Testing if Indicated for Diagnosis or Safety • Body Composition • Electrocardiogram • Recommended especially if using anti-obesity medications • Resting metabolic rate (RMR) • Sleep study • Imaging studies of the liver • Coronary calcium score • Echocardiogram, cardiac stress test • Ankle-brachial index • Anaerobic threshold/VO2 testing • Further cortisol testing if Cushing’s syndrome is suspected • Further hormonal testing if hypogonadism is suspected • If a structural lesion of the pituitary/hypothalamus is suspected (e.g., craniopharyngioma, pituitary tumor, etc.): magnetic resonance imaging or computed tomography of the brain

86 Summary: Putting it All Together

• The history, physical examination, and initial testing of the patient affected by obesity has unique aspects that enable the medical practitioner to:

• Create a supportive environment free of stigma or bias • Perform a comprehensive assessment and evaluation to develop an individualized treatment plan to optimize patient success • Each patient is unique, and so the approach to treating each patient is unique

87

Treatment Options

Assessment and Evaluation

Nutrition Surgery/ Procedures

Medication Behavior Physical Activity

88 Polling question

A 45-year-old man with a BMI of 38 kg/m2 has bariatric surgery undergoing a gastric sleeve procedure. He notices an immediate reduction in hunger. What hormonal change is most likely contributing to his reduction in hunger?

• Decrease in CCK • Decrease in ghrelin • Decrease in PYY • Increase in insulin • Increase in leptin

89

References

• Alberti, K. G. M. M., et al. "Harmonizing the metabolic syndrome." Circulation 120.16 (2009): 1640-1645. • Bell, Joshua A., Mika Kivimaki, and Mark Hamer. "Metabolically healthy obesity and risk of incident type 2 diabetes: a meta-analysis of prospective cohort studies." Obesity reviews 15.6 (2014): 504-515 • DeFronzo, Ralph A. "From the triumvirate to the ominous octet: a new paradigm for the treatment of type 2 diabetes mellitus." Diabetes 58.4 (2009): 773-795. • Diabetes Prevention Program Research Group. "Reduction in the incidence of type 2 diabetes with lifestyle intervention or metformin." N Engl j Med 2002.346 (2002): 393-403.

93 References

• Diabetes Prevention Program Research Group. "Long-term effects of lifestyle intervention or metformin on diabetes development and microvascular complications over 15-year follow-up: the Diabetes Prevention Program Outcomes Study." The lancet Diabetes & 3.11 (2015): 866-875. • Hansen, Louise, et al. "Metabolically healthy obesity and ischemic heart disease: a 10-year follow-up of the Inter99 study." The Journal of Clinical Endocrinology & Metabolism 102.6 (2017): 1934-1942. • Hoskin, Mary A., et al. "Prevention of Diabetes Through Lifestyle Intervention: Lessons Learned from the Diabetes Prevention Program and Outcomes Study and its Translation to Practice." Current nutrition reports 3.4 (2014): 364-378. • Williamson DF. “Descriptive epidemiology of body weight and weight change in U.S. adults”. Ann Intern Med. 1993;119(7 Pt 2):646–9

94 Fundamentals of Obesity Treatment:

Nutrition

46 Objectives

• Review nutrient labeling • Identify the characteristics of different types on nutrients • Introduce the hormonal concept of energy balance • Review common dietary interventions

*Disclaimer: The field of nutrition is complex and hotly debated. This lecture will only cover basic concepts of nutrition.

96

What is Nutrition?

The process of obtaining the food necessary for health and growth.

What IS necessary for health and growth? • Macronutrients • Micronutrients • Water • Other non-nutritive substances

97 Nutrients

Substances obtained from food that promote proper function • Have at least one or more of the following functions: • Provide energy • Provide structural material • Participate in regulating body processes Categories of nutrients Macronutrients: • Protein, carbohydrate, fat (alcohol provides energy but is not a macronutrient, since it is not essential for body functioning) Micronutrients: • Vitamins, minerals, trace elements

Fahey, Insel, Roth, Chapter 8 Nutrition - McGraw-Hill Higher 98 Education

Food Provides Energy

• All foods can be oxidized to release energy • Energy stored in the chemical bonds is released from carbohydrates, fat, and protein, and the liberated energy is used to synthesize ATP • Energy content in foods is measured in kilocalories and is commonly referred to as just “calories” • Definition of a kilocalorie is the amount of energy required to raise the temperature of 1 gram of H2O by 1°C • Measurement is based on combustion not metabolism

Fahey, Insel, Roth, Chapter 8 Nutrition - McGraw-Hill Higher Education

99 Definitions

• RDA: Recommended Dietary Allowance – RDA is the average daily dietary intake of a nutrient that is sufficient to meet the requirement of nearly all healthy persons • AI: Adequate Intake – Established when an RDA can not be determined. A nutrient has either RDA or AI – Based on observed intake in a group of healthy individuals • UL: Tolerable Upper Intake Level – The highest intake of a nutrient that is likely to pose no risk of toxicity for almost all individuals • EAR: Estimated Average Requirement – Amount of a nutrient that is estimated to meet the requirement of half of all healthy individuals in the population • DV: Daily Value (based on 2,000 calories / day)

100 https://www.nal.usda.gov/fnic/macronutrients. Accessed 11-28-2017.

Acceptable Macronutrient Distribution Range (AMDR)

• Acceptable macronutrient distribution range • Expressed as a percentage of total calories • Note that this is a standardized definition – it may not be ideal for all patients, and was not recommended with weight loss in mind RDA % of total calories Macronutrient (recommended (adults) dietary allowance) Protein 10-35% 56 gm/d (men), 46 (women) Carbohydrate 45-65% 130 gm/d Fat 25-35% 30 gm/d

101 https://www.nal.usda.gov/fnic/macronutrients. Accessed 11-28-2017. Macronutrients

Macronutrient Kcal / gm Carbohydrates 4 Fat 9 Protein 4 Water 0

Alcohol 7 (not a macronutrient per se)

https://www.nal.usda.gov/fnic/macronutrients. Accessed 11-28-2017.

102

Protein

103 Dietary Protein

Complete Proteins - complete balance of essential amino acids Animal proteins are complete proteins Plant derived complete proteins: soy, quinoa, and amaranth Vegetarian diets - food combinations may be required to assure consumption of all essential amino acids Incomplete Proteins - missing one or more essential amino acid Examples: – Some grains – Legumes – Gelatin

th 104 Contemporary Nutrition, 7 Ed., GM Wardlaw, A M. Smith, 2009 Academy of Nutrition and Dietetics.

Protein and Weight Loss

• Evidence exists that proteins exert an increased thermic effect of food • Evidence is also growing that higher protein diets increase satiety when compared to lower protein diets • Higher protein intake may increase weight loss and possibly increase percentage of fat loss • Dietary protein helps maintain lean mass during weight loss, which is directly proportional to resting energy expenditure • Protein deficiency: protein alone - Kwashiorkor (big belly, edema) protein and calorie deficiency – Marasmus (big head, skinny body)

105 Halton TL. J Am Coll Nutr. 2004. How Much Protein Do You Need?

• The Institute of Medicine recommendation – Minimum of 0.8 grams of protein/kg/day – Higher in children, pregnancy and lactation – Based on ideal body weight

• Optimal protein intake should consider – Quality of the protein – Body composition – Physiologic state (growth, illness, injury, sarcopenia) – Level of energy intake / physical activity

Dietary reference intakes for energy, carbohydrate, fiber, fat, fatty acids, cholesterol, protein, and amino acids (macronutrients). Institute of Medicine, National Academy Press, 2001; Soenen S et al. J. Nutr, 2013; 143(5); Tang M et al. Obesity, 2013, 21(3); Blackburn G. Postgrad Med J, 1984, 60: S3. 106 Steelman GM, Westman EC (eds). Obesity: Evaluation and Treatment Essentials. London: Informa UK Ltd., 2010.

Protein and Weight Loss

Protein Amount

Maintenance .7 – 1 g/kg/d

1.2-1.5 g / kg lean body Weight loss weight ~90-120 g / d

107 Steelman GM, Westman EC. Obesity: Evaluation and Treatment Essentials. 2010. Fat

108

Why We Need Fat

• Fats or lipids are a diverse group of compounds that are insoluble in water • Digestion and absorption of fat results in free fatty acids and glycerol • Lipid function - energy source and important in: • Immune response (omega-3 fatty acids) • Cell membrane structure (phospholipids) • Brain tissue (cerebrosides) • Synthesis of bile acid, vitamin D, steroid hormones

109 Fatty Acids

• Monounsaturated fatty acids – Have one double bond – Higher levels in olive, nuts, seeds, and avocados • Polyunsaturated fatty acids (PUFAs) – Have more than one double bond – Protective role in coronary artery disease and inflammatory diseases. – Essential fatty acids • Linoleic acid (omega-6) & Alpha-linoleic acid (omega-3) • Insufficient consumption results in essential fatty acid deficiency • Precursors to many vasoactive and immunologic compounds including prostaglandins and eicosanoids.

110

Fats

• Saturated, Trans-fats, Cholesterol

– No required role other than as an energy source (in adults)

– Saturated fat: animal fats, coconut and palm kernel oil

– Cholesterol: liver, eggs

– Trans-fats – partially hydrogenated vegetable oil

• stick margarine, vegetable shortening

• Conjugated linoleic acid (CLA) is a naturally occurring trans fat not shown to be detrimental to health

https://www.nal.usda.gov/fnic/macronutrients. Accessed 11-28-2017. 111 Other Fatty Acids

• Synthetic trans-FA linked to cardiovascular disease and malignancies. • Naturally occurring trans-FA are not associated with disease • The American Heart Association recommends that trans fatty acids and saturated fat make up no more than 10% of fat calories. • New evidence suggests that saturated fat and even total fat grams may not be the issue in heart disease. • The FDA banned trans-FA in 2018.

Know Your Fats. AHA. 2008. 112 http://annals.org/aim/article/1846638/association-dietary-circulating-supplement-fatty-acids-coronary-risk-systematic-review accessed 7/11/2017

Carbohydrates

113 Carbohydrates (CHO)

• Carbohydrates are the most abundant organic molecules on earth. • They are an important source of fuel as well as being an important structural element. • They are not an essential macronutrient, as the liver and kidney can synthesize glucose.

114 Steelman GM, Westman EC. Obesity: Evaluation and Treatment Essentials. 2010.

Fructose

• Substantial increase in the amount of dietary fructose consumption since 1970’s – Fructose in nature is primarily from fruits and some vegetables – Sucrose (table sugar) is 50% fructose, 50% glucose (disaccharide) – High-fructose corn syrup (HFCS) is 55% fructose; 45% glucose (monosaccharide mix) – Pure fructose can be part of a diet, but more than 30 grams (6 teaspoons) a day may be too much • 124 lbs. of sweetener/person/year in 2018 of caloric sweetener (mostly sugar/HFCS) • The largest single source is from added sugar

– Desserts, candies, soft drinks and much, much, more

Buzby JF. USDA. 2005 115 https://www.ers.usda.gov/data-products/sugar-and-sweeteners-yearbook-tables.aspx. Fructose

• Liver only site for fructose metabolism • Low level fructose intake → Glucose • High level of fructose intake → De Novo Lipogenesis – ↑ FFA, triglycerides and uric acid • Excess dietary consumption of fructose, may contribute to insulin resistance and to metabolic syndrome.

Basciano H. Nutr Metab (London). 2005. 116 Havel PJ. Nutr Rev. 2005.

Fiber

• Non-digestible carbohydrates

– Improves laxation

– Reduces risk of CHD

– Assists in maintaining normal blood glucose levels

117 https://www.nal.usda.gov/fnic/macronutrients. Accessed 11-28-2017. Vitamins

• Most vitamins cannot be synthesized in our bodies - must be ingested; essential for maintenance of normal metabolic functions Water soluble vitamins are not significantly stored - must be provided regularly; excesses excreted in urine Fat soluble vitamins are better stored and excesses can accumulate to toxic levels • “Natural” and synthetic vitamins have similar kinetics • Considerable controversy exists but most feel it might be good idea to take a multivitamin during weight loss • Vitamin D bioavailability may be decreased in obesity

118 Graat JM. JAMA. 2002; Wortsman J, AJCN 2000.

Sugar Alcohols

• Common replacement item for sugar Sugar Alcohol – Carbohydrate that is neither a sugar nor an alcohol Kcal/gm – Incompletely absorbed and metabolized by the body Sorbitol 2.6 – Calorie content range from 0 to 3 kcal/gm Xylitol 2.4 – AKA polyols, they have a limited insulin response Maltitol 2.1 – Polyols have a limited effect on glucose levels Isomalt 2.0 – Since sucrose contains 4 kcal/gm, the reduction in Mannitol 1.6 calories may be utilized to calculate caloric and Erythritol 0 carbohydrate intake • The terms “net carb” and “low carb” are not defined by the FDA – Non-absorbed polyols can have a laxative effect

119 https://foodinsight.org/sugar-alcohols-fact-sheet/ Accessed 12-26-2019. Other substances that may reduce carbohydrate and caloric impact

• Prebiotics: generally defined as a “nondigestible food ingredient that beneficially affects the host by selectively stimulating the growth and/or activity of one or a limited number of bacteria in the colon”. Examples of prebiotics include inulin, oligosaccharides derived from sucrose, galactose, and xylose. (Hutkins, Current Opin Biotechnol 2015)

• Resistant starch: defined as a starch that passes into the large intestine not absorbed not digested by the stomach or small intestine. Resistant starches may be modulate colonic microbial flora, and fermentation products include short chain fatty acids such as butyrate. Evidence suggests that resistant starches may improve glycemia and insulin sensitivity in patients with type 2 diabetes. (Gao, Lipids Health Dis 2019)

120

Electrolytes, Minerals and Trace Elements

• Three particular elements could be relevant to obesity medicine: • Macro Minerals: >100 mg/day • Na, K, Fe, Ca, Mg, P • Micro Minerals: <100 mg/day • Se, I, Cr, Zn, Cu • Certain diets can cause electrolyte imbalance (Na, K, Cl) • Micro minerals are frequently components of enzymes • Recommended Dietary Allowances (RDAs) have not been established for all of them

121 Srivastava AK. Diabetes MedicinWang, ZQ. Eur J Clin Nutr. 1996. e. 2005. Nutritional Approach Concepts

122

Nutritional interventions for the treatment of excess weight and obesity-related disease should be viewed as part of a comprehensive lifestyle intervention rather than a “diet”

Obesity is a chronic relapsing disease requiring lifelong changes

123 Nutritional therapy usually focuses on manipulation of calories as well as the quality and quantity of the three macronutrients:

Carbohydrates Protein Fat

124

Energy Balance versus Body Weight Regulation

• Classic thinking: Energy balance is a function of ‘energy in’ versus ‘energy out.’ Calorie intake versus expenditure determines amount of fat mass.

• Hormonal concept: Energy storage is controlled by hormonal signals that regulate fat storage (lipogenesis) versus fat burning (lipolysis).

125 Macronutrient Metabolism

• Stimulates insulin production → promotes fat storage Carbohydrate • Variable rate of conversion to glucose → Glycemic Index • Variable effect on satiety

• Slower rate of digestion = greater satiety Protein • Limited effect on insulin production • Higher thermal effect of food

• Does not stimulate insulin production Fat • More satiating • Highest caloric content

126

Factors to Consider

“All diets work, all diets fail”

Patient adherence best predictor of long-term success

Dansinger, JAMA 2005

127 Individual Factors to Consider

Patient Medical Nutritional Preference Conditions Requirements • Personal • Diabetes •Exercise • Ethnic • Hypertension •Growth • Work schedule • Dyslipidemia • Food • Religious • Liver disease intolerance • Knowledge • Kidney disease • Celiac disease • Gout

128

The Glycemic Index

• The GI is a measurement of the rise in blood glucose that occurs after the ingestion of food • The glycemic index (GI) of a food affects how much insulin is secreted in response to consuming that food • Foods with a lower glycemic index are absorbed more slowly and lead to a lower glucose excursion and insulin response

129 Jenkins DJ, et al. Am J Clin Nutr, 1981, 34(3). Carbohydrate-insulin Model

• Food selection affects insulin production to varying degrees • High glycemic index foods produce a greater insulin response • High levels of circulating insulin increases fat storage and suppresses fat burning (lipolysis) • Consuming fewer calories than your body requires (negative energy balance) decreases insulin production • Consuming a low carbohydrate diet decreases insulin production • Low insulin levels promotes fat release and fat burning

130

Carbohydrate-insulin Model

Take home message:

• Many metabolic conditions associated with obesity are mediated through insulin resistance and hyperinsulinemia • Consuming a diet that minimizes the amount of insulin in circulation is likely to be beneficial for weight loss and metabolic disease

131 Hormonal Regulators

• Diet impacts hormonal regulators of appetite – Sufficient protein intake can improve satiety – Higher carbohydrate intake may increase appetite – Hormonal changes in weight loss favor weight regain • Increase in ghrelin • Decrease in leptin • Decrease in CCK, PYY and other agents of satiety • Hormonal changes persist for at least 1 year

Sumithran NEJM 2011 132 Gosby A, Ob Rev 2013 Greenway F, IJO 2015

2015-2020 Dietary Guidelines

1. Follow a healthy eating pattern across the lifespan 2. Focus on variety, nutrient density, and amount 3. Limit calories from added sugars and saturated fats and reduce sodium intake 4. Shift to healthier food and beverage choices 5. Support healthy eating patterns for all

133 http://health.gov/dietaryguidelines/2015/guidelines/executive-summary/ Interpreting the nutrition label

1. Check the serving size. 2. Look for calories per serving, and calculate calories based on your serving size 3. Assess the macronutrient (protein, carbohydrate, fat) content per serving 4. Check fiber, sugar, and added sugar per serving 5. Analyze the micronutrient content, especially if sodium, potassium, or other nutrient needs to be adjusted for health concerns. 6. Look at the ingredients. Check out additives, such as the presence of trans fats.

134

Old New Label Label

Change http://www.fda.gov/Food/Guidance Delayed Regulation/GuidanceDocumentsR egulatoryInformation/LabelingNutri tion/ucm385663.htm#highlights

135 Specific Dietary Options

136

Some Types of Diets

Diet Type Characteristics

Calorie Deficit Calorie intake below estimated energy expenditure

Low Carbohydrate <45% calories from carbohydrates

Low Fat <20% calories from fat

Low Glycemic Index Limit foods with high-glycemic index or high-glycemic load Fish, nuts, healthy fats, fruits, vegetables, whole grains, Mediterranean legumes. Limit meat and dairy DASH Dietary approaches to stop hypertension

Meal Replacement Pre-packaged foods – avoids selection and preparation

Vegetarian Exclusion of all or most animal products 137 Katz DL et al. Ann Rev Pub Health;35:83-103. Health Benefits of These Diets

Diet Type Potential Health Benefits

Calorie Deficit  weight

Low Carbohydrate  trig,  HDL

Low Fat  total cholesterol and  LDL

Low Glycemic Index Maintaining weight loss

Mediterranean  CV mortality and  DM II

DASH  blood pressure

Meal Replacement  weight

Vegetarian  LDL,  CV mortality and  DM II 138 Katz DL et al. Ann Rev Pub Health;35:83-103.

Calorie Deficit Diets

• Defined by calorie intake below estimated energy expenditure to create a calorie deficit and weight loss • A calorie restricted diet should contain adequate protein and other essential nutrients for maintaining good health • Calorie goal may or may not be combined with reduction of specific macronutrients (eg low fat or low carb) or with other dietary patterns (eg reduced calorie Mediterranean diet) • Low calorie diets are usually prescribed to achieve a specific calorie deficit (eg 500- 1000 calories below estimated or measured daily energy needs) • Severely energy restricted diets may be prescribed to achieve greater weight loss

139 Bray G et al. Handbook of Obesity. 2008: 269-290. www.joslin.org. Accessed 7-26-2015 Calculations for Calorie Needs

• Mifflin St Jeor – Men: 10 X Weight (kg) + 6.25 X Height (cm) - 5 X Age (yrs)+ 5 – Women: 10 X Wt (kg) + 6.25 X Height (cm) – 5 X Age (yrs) – 161

• Harris Benedict – Men: 66.5 + 13.75 X Weight (kg) + 5.003 X Height (cm) – 6.775 X Age (yrs) – Women: 655.1 + 9.563 X Weight (kg) + 1.85 X Height (cm) - 4.676 X Age (yrs)

• Sedentary = BMR x 1.2 (little or no exercise, desk job) • Lightly active = BMR x 1.375 (light exercise/ sports 1-3 days/week) • Moderately active = BMR x 1.55 (moderate exercise/ sports 6-7 days/week) • Very active = BMR x 1.725 (hard exercise every day, or exercising 2 x/day) • Extra active = BMR x 1.9 (hard exercise 2 or more times per day, or training for marathon, or triathlon, etc.

140

Low-carbohydrate (CHO) Diets

• CHO (Carbohydrate amount controlled; refined carbohydrates eliminated) • Reduces insulin; blood glucose controlled • Examples: Zone (30% Fat; 40% CHO; 30% Protein), South Beach (50-150 g CHO), VLCD • If < 50g CHO: ketogenic (Atkins) • Joslin Diabetes Center: new nutrition guidelines T2DM – ~40% CHO: low GI, high fiber – 20-30% protein – 30% fat (no trans fat, 7-10% sat fat, 20% mono and poly unsaturated fats)

141 Bray G et al. Handbook of Obesity. 2008: 269-290. www.joslin.org. Accessed 7-26-2015 Low Fat Diets

• <20% fat, 55-65% carb, ~15% protein • <10% saturated fat EXPECTED BENEFITS

• Reduction in total cholesterol and LDL

• Recommended by American Diabetes Association, American Heart Association, American Cancer Society. Recent recommendations include limiting added sugars

Bray G. Handbook of Obesity. 2008; 269-290. 142 http://circ.ahajournals.org/content/circulationaha/early/2015/08/04/CIR.00000000000 00230.full.pdf accessed 7/11/2017

Mediterranean Diet

• 35-40% fat • High in monounsaturated fat and high in Omega-3 – Extra virgin olive oil is the primary fat • Whole grains, fruits, vegetables, legumes, nuts, monounsaturated oil (olive oil, avocados, etc.) • Fish and seafood, moderate alcohol • Limited poultry, dairy, red meat infrequent

143 Mediterranean Diet

• PREDIMED Study 31% lower risk of first cardiovascular event* 52% lower risk of developing type 2 diabetes* • Lyon Heart Study 50%-70% lower risk of recurrent CHD

*PREDIMED Study for CVD was revised in 2018 with similar outcome

Estruch R. N Engl J Med. 2013. 144 de Lorgeril M. Circulation. 1999.

Diet and Blood Pressure: DASH Diet

• Dietary approaches to – Low sodium (<1500 mg/d) stop hypertension – Whole grains (6-8 servings/d) – Non-starchy veggies (4-5 /d) • 2 weeks, drops blood – Fruits (4-5 /d) pressure 8 - 14 points – Dairy (2-3 low fat or fat free / d) • Not intended for weight – Lean meat, fish, poultry (6 /d, try for more fish and less red meat) loss (unless caloric intake is restricted) – Nuts, seeds, legumes (4-5 s/wk) – Fats/oils (2-3 s/d, avoid saturated and trans) – Sweets (5 or less / week) – ETOH: men ≤ 2/d, women ≤ 1

Mayo Clinic Staff. Dash. 2013. 145 Meal Replacements

• Isocaloric energy-restricted meal replacements for 2 meals / day provide greater weight loss than diets composed of conventional foods • Similarly in weight-loss maintenance, but with 1 meal replacement / day • Can be used with any dietary approach • Partial MR plans produce greater weight loss than equivalent calorie diets • Weight loss doubled compared to traditional plans • Portion and calorie controlled; improved nutrition

ADA. J AM Diet Assoc. 2009. Flechtner-Mors M et al. Obesity Res. 2000. Heymsfield SB et al. Int J Obesity. 2003. Tsai AG, Wadden TA. Obesity 2006;14(8):1283-1293. 146 Li Z, Bowerman S, Heber D. Obes Manag. 2006;2(1): 23-28.

Meal Replacements

• Can be used as a complete meal plan or as meal substitutes for one or two meals • Have been shown to be successful – Used successfully in the “Action for Health in Diabetes” Program (Look AHEAD) – In the first-year, exercise, attendance at treatment session, and use of meal replacements showed the highest correlation with weight reduction

147 Very Low Calorie Diet (VLCD)

• Often defined as 400-800 kcal/day; often implemented by use of commercially prepared formulas • By nature this diet is both low-carb and low-fat • Protein needs must be met - 1.2-1.5 g/kg of ideal body weight • Proper medical training is required – EKG monitoring – Lab / Electrolyte / Renal Function / Liver Function – Medication titration – monitor for dehydration, low blood pressure, hypoglycemia • Meta-analysis of published data does not show superior long- term weight loss as compared with other strategies

Tsai G & Wadden T. “The Evolution of VLCD’s: Update and Metanalysis.” Obesity 2006 Vol 14 No. 8; 1283 – 1293. 148

Vegetarian / Vegan Diets (Low Fat)

• Vegetarian Categories – Vegans: no animal products of any type – Lacto-ovo: eat dairy and eggs – Pescatarian: eat fish but no red meat or poultry • Protein sources: low-fat dairy, soy, legumes, whole grains, nuts, seeds, etc.

McEvoy C et al. Nutrition & Health: Nutrition Guide for Physicians. 2010. Barnard ND et al. J Acad Nutr Diet. 2015 (in press) Jenkins DJA et al. BMJ Open 2014(4). 149 Yokoyama Y et al. Cardiovasc Diagn Ther. 2014;4(5):373-382 Vegetarian / Vegan Diets (Low Fat)

• Expected benefits – Weight loss – May lower risk of certain cancers including colon cancer, cardioprotective – Decreases LDL, decreases DM risk, improved glycemic control

• Potential risks

– Increased risk of B12, iron, zinc, calcium, vitamin D and omega-3 deficiency

– May need to supplement B12, vitamin D and omega-3

150

Intermittent Fasting for Weight Loss

• What is it? – Cycles between periods of fasting and unrestricted eating – Time-restricted feeding: designated daily time frame for fasting, typically 12-18 hours per 24 hour period – Alternate-day fasting: alternate eating days and fasting days. On fasting days, have one meal that provides ~25% of daily caloric needs – Whole-day fasting: 1-3+ contiguous days of full fasting, may include one meal

151 https://www.hsph.harvard.edu/nutritionsource/healthy-weight/diet-reviews/intermittent-fasting/ Accessed 8-19-18 Intermittent Fasting (IF): Does it work? •Animal data indicate benefits for health and aging • Human studies show improvement in: – Obesity – Insulin resistance – Dyslipidemia – Hypertension – Inflammation

de Cabo, R., & Mattson, M. P. (2019). Effects of Intermittent Fasting on Health, Aging, and Disease. 152 New England Journal of Medicine, 381(26), 2541-2551.

Post-bariatric Surgery

Liquid diet • First 2 weeks post-op • Hydration first, then work on protein • Absolutely NO carbonated beverages Pureed diet • 2-4 weeks post-op Transition to “real food” • 4-8 weeks post-op • Tiny amounts Long term • Small plate size • Protein first, then colorful vegetables/salad Concept of eating: “How little can I eat and be satisfied?” NOT “How much can I cram in there?”

153 Post-bariatric Surgery

Patients need to be monitored for surgical complications and nutritional deficiencies.

Refer to American Society for Metabolic and Bariatric Surgery's Clinical Practice Guidelines (www.asmbs.org).

154

Diet Treatment Summary

Obesity is a chronic disease Dietary changes need a long-term strategy - Weight loss phase - Maintenance phase

155 Polling Question A 50-year-old man weighs 75 kg. What is his minimum daily protein intake requirement?

• 30 grams of protein • 60 grams of protein • 75 grams of protein • 100 grams of protein • 112.5 grams of protein

156

Polling Question

A 24-year-old female recently gained 20 lbs. and relates that she has been working late and eating a lot more fast food. Which of the following statements is accurate and supports the reason for her weight gain?

•Fast foods with a higher glycemic index produce a lesser insulin response • High fat, low carb intake is reducing total energy expenditure • Increased carbohydrate intake is increasing appetite • Increased protein intake is reducing satiety

159 Further Educational Resources

1. Creating a Meal Plan for Patients with Obesity: Ethan Lazarus, MD, FOMA 2. Nutrition Strategies: The Mediterranean and Ketogenic Diets: Michael Ozner, MD, FACC, FAHA and Andreas Eendfeldt, MD 3. Using Meal Replacements in Obesity Treatment: Grade School to Grandparents: Angela Fitch, MD, FACP, FOMA 4. Eating Patterns for Weight Management: Marie Pierre St-Onge, PhD, FAHA 5. Download the Obesity Algorithm at ObesityAlgorithm.org

www.OMAcademy.org 162 Fundamentals of Obesity Treatment:

Physical Activity

79 Objectives

• Recognize the benefits of physical activity • Understand the fundamentals of physical activity used in the treatment of obesity • Identify the latest Physical Activity Guidelines • Describe the components of an exercise prescription

165

Physical Activity to Improve Health

Adiposopathy (Sick Fat Disease) • Improve body composition • Possibly improve adipocyte function (“train” fat cells) ‒ Improve insulin sensitivity ‒ Increase mitochondrial biogenesis ‒ Increase browning (“beiging”) of fat cells

Non-adipose Parameters • Improve metabolic health • Improve musculoskeletal health • Improve cardiovascular health • Improve pulmonary health • Improve mental health (e.g., mood, happiness, sense of well-being) • Improve sexual health 166 Obesity Algorithm®. ©2017-2018 Obesity Medicine Association. Reference/s: [186] [187] [188] [189] Physical Activity Improves Health

• Improves cardiovascular function • Improves blood glucose regulation • Improves dyslipidemia • Increases HDL cholesterol levels • Lowers blood pressure • Improves depression & anxiety • Lowers risks of some cancers • Lowers risk of all-cause mortality

Image property of the Canadian Obesity Network

Johannsen et al Sports Med 2016; 46(12):1809-1818 Ford et al. Prev Med 2012 Jul 55(1):23-27 167 Warburton et al Curr Opin Card; July 2015

Physical Activity Improves Health

• Lowers risk CVD mortality • Lowers risk of falls (older adults) • Improves cognition • Improves weight status • Improves sleep • Improves physical function • Improves bone health • Improves quality of life Image property of the Canadian Obesity Network

Johannsen et al Sports Med 2016; 46(12):1809-1818 Ford et al. Prev Med 2012 Jul 55(1):23-27 168 Warburton et al Curr Opin Card; July 2015 How Much Physical Activity Is Enough?

• General health benefit Moderate intensity aerobic physical activity 150 minutes weekly (30 min 5 X per week+ 2-3 days with strength training)

• Weight loss and weight maintenance May require ≥ 300 minutes of moderate intensity aerobic physical activity + strength training

“Combining both caloric restriction and physical activity tendstobemostbeneficial for weightlossratherthanjustcaloric restrictionorjustphysical activity”.

. 169 U.S. Health and Human Services. 2018.

Risks of Sedentary Behavior

More time spent in sedentary behavior increases risk of: • All-cause mortality • Cardiovascular disease mortality • Cardiovascular disease • Type 2 diabetes • Cancer of the bladder, breast, colon, endometrium, esophagus, kidney, lung, and stomach

U.S. Health and Human Services. 2018

170 Physical Activity Guidelines for Adults

• Move more and sit less • Any amount of physical activity has health benefits

171 U.S. Department of Health and Human Services. Physical Activity Guidelines for Americans, 2nd edition. Washington, DC: U.S. Department of Health and Human Services; 2018.

Physical Activity Guidelines for Adults

Spread aerobic activity throughout the week Even short bouts of physical activity (10 minutes) can have health benefits Muscle strengthening activities (moderate or greater) 2+ days/week Additional health benefits for 300 + minutes/wk moderate intensity physical activity

U.S. Department of Health and Human Services. Physical Activity Guidelines for Americans, 2nd edition. Washington, DC: 172 U.S. Department of Health and Human Services; 2018. Physical Activity Guidelines for Adults

General Health Benefit 150 – 300 minutes per week of moderate intensity aerobic activity OR 75 - 150 minutes per week of vigorous intensity aerobic activity OR An equivalent combination of moderate-vigorous aerobic activity

U.S. Department of Health and Human Services. Physical Activity Guidelines for Americans, 2nd edition. Washington, DC: 173 U.S. Department of Health and Human Services; 2018.

Physical Activity Guidelines for Adults

Substantial Weight Loss (>5%) May require 300 + minutes/week moderate intensity physical activity

U.S. Department of Health and Human Services. Physical Activity Guidelines for Americans, 2nd edition. Washington, DC: 174 U.S. Department of Health and Human Services; 2018. Physical Activity Guidelines for Children

• Key Guidelines for Preschool-Aged Children – Age 3 through 5 • Be physically active throughout the day to enhance growth and development. • Encourage active play that includes a variety of activity types.

U.S. Department of Health and Human Services. Physical Activity Guidelines for Americans, 2nd edition. Washington, DC: 175 U.S. Department of Health and Human Services; 2018.

Physical Activity Guidelines for Children

• Key Guidelines for Children and Adolescents – Children and adolescents ages 6 through 17 years • 60 minutes or more of moderate-to-vigorous physical activity daily: • Aerobic: including some vigorous-intensity physical activity on at least 3 days a week. • Muscle-strengthening: including muscle-strengthening physical activity on at least 3 days a week. • Bone-strengthening: including bone-strengthening physical activity on at least 3 days a week.

176 U.S. Department of Health and Human Services. Physical Activity Guidelines for Americans, 2nd edition. Washington, DC: U.S. Department of Health and Human Services; 2018. Physical Activity Guidelines for Children Children and Adolescents – Health Improvements • Improved bone health (ages 3 through 17 years) • Improved weight status (ages 3 through 17 years) • Improved cardiorespiratory and muscular fitness (ages 6 through 17 years) • Improved cardiometabolic health (ages 6 through 17 years) • Improved cognition (ages 6 to 13 years)* • Reduced risk of depression (ages 6 to 13 years)

U.S. Department of Health and Human Services. Physical Activity Guidelines for Americans, 177 2nd edition. Washington, DC: U.S. Department of Health and Human Services; 2018.

What is the State of Physical Activity in the US?

• U.S. adults 18 years of age and over who met the 2008 federal physical activity guidelines for:

Aerobic Physical Activity: 49.0%* Muscle Strengthening Activity: 23.6%** Both Aerobic + Strengthening activity: 20.9%*

ONLY 1 in 5 are meeting the PA Guidelines!

*Centers for Disease Control and Prevention. Exercise or Physical Activity 2015. **Centers for Disease Control and Prevention. Exercise or Physical Activity 2013. 178 Getting Beyond the 150min/wk Health Benefit Number

Let’s look at the data on Physical Activity In Weight loss & Weight Maintenance

179

Weight Loss and Physical Activity

• Relationship between mean amount of weight change & physical activity • Physical activity: – NOT the major contributor to active weight loss, but changes in body composition do occur – 3 kg or 6.6 lbs of weight loss • 8 months • 20 miles of jogging/week – BUT 4.8 kg or 10.5 lbs of FAT loss

180 Slentz CA. Arch Intern Med. 2004. Physical Activity is ESSENTIAL in Weight Maintenance

Concomitant Behavior Therapy Weekly Biweekly Monthly 0 -2 -4 -6 <150 min/wk -8 -10 -12 -14 -16 >150 min/wk *P<0.05 >200 min/wk Change in Weight (kg) Weight Change in 061218 Time (months)

181 Jakicic JM. JAMA. 1999.

Look AHEAD Successful Maintainers and Physical Activity

Successful maintenance of weight loss at 4 years required:

2000kcal/wk of PA = 60-70 minutes/day = Approx. 420 moderate intensity PA minutes/week

182 Wadden TA. Obesity. 2011 Avoiding Weight Regain

What do we know about SUCCESSFUL maintainers?

National Weight Control Registry Data – 78% eat breakfast every day – 75% weigh themselves at least once a week – 62% watch less than 10 hours of TV per week – 90% exercise, on average, about 1 hour per day – Maintainers expend an avg. of 2000kcal/wk in physical activity

Wyatt et al. Obes Res 2002 Feb;10(2):78-82 Butryn et al. Obestiy 2007 Dec;15(12) 3019-6. 183 Raynor et al. Obestiy 2006 Oct;14(10) Phelan et al. Obesity 2006 Apr;14(4):710-6 .

NEAT: All Physical Activity Is Not Exercise!

Non-Exercise Activity Thermogenesis • Activities that are NOT sleeping, eating, or exercise • Includes common daily activities like fidgeting, walking, standing, and climbing stairs • Up to 2000 kcal of energy expenditure per day • Decreases CV mortality and improves metabolic parameters like lipid and insulin levels • Hypothalamus appears to regulate NEAT

184 Villablanca et al. Mayo Clin Proc 2015: Apr 90(4) 509-19. Who Should Provide Physical Activity Counseling?

The U.S. Preventive Services task force believes it is the health care provider’s role to screen and provide (or at a minimum, refer) this counseling when treating patients affected by overweight or obesity.

• The U.S. Preventive Services task force recommends screening all adults for obesity. Clinicians should offer or refer patients with a body mass index (BMI) of 30 kg/m2 or higher to intensive, multicomponent behavioral interventions. • B recommendation • USPSTF updated statement released Sept 2018 – no change in recommendations

185 U.S. Preventive Services Task Force. Released June 2012. Accessed July 2017

Patients Want to Hear It!

“65% of patients would be more interested in exercising to stay healthy if:

Advised by their doctor AND Given additional resources”

- American College of Survey Image property of the Canadian Obesity Network

186 Physical Activity & The 5 A’s of Obesity Treatment

Helping patients find the motivation and confidence to become physically active!

ASK ASSESS ADVISE AGREE ASSIST

187 Vallis et al. Can Fam 2013 Jan: 59(1):27-31

Optimizing the Exercise Prescription Using 5 A’s

• Ask: Let’s develop an Exercise • Agree on realistic goals Prescription (Ex Rx) together! • Agree on FITTE for a defined • Assess current level of physical and specified interval. activity and readiness for change • “In the next week I can walk on the treadmill 2 times for 30 minutes at 3.0 mph” • Assess resources • Assist with the “optimal default” • Assess potential barriers or back up plan including general health and physical limitations • Assist with strategies for lapse/relapse • Advise on any pre-physical activity diagnostic testing 188 The Question Is….

Do you know what to say?

189

Core Components of an Exercise Prescription

FITTE

Frequency Intensity Time Type Enjoyment

190 Writing an Exercise Prescription • Aerobic activity • Strength training

– Type – Two days per week – Frequency – All major muscle groups – Day and time – 8-12 reps – Intensity –Examples: –Time • Bodyweight exercises – Steps per day (push-ups, lunges, etc) – Make it fun • Carrying heavy loads • Heavy gardening

191

Assess Mobility

Unable to Walk Limited Mobility, No Substantial Limitations Able to Walk to Mobility • Seated exercise program • Arm exercises (i.e., arm cycling) •Walking • Swimming/aquatic exercises (e.g., • Exercise/physical activity • Swimming/aquatic exercises (e.g., shallow or deep water exercises) prescription plan driven by patient shallow or deep water exercises) • Gravity-mediated physical activity and guided by clinician • Gravity-mediated physical activity • Consider • Assess for special equipment • Assess for special equipment evaluation needs needs ‒ Recommend rehabilitation & physical therapy guided activity program ‒ Set physical activity goals ‒ Assess special equipment needs

192 Obesity Algorithm®. ©2017-2018 Obesity Medicine Association. Reference/s: [191] [192] Getting Started……Not Every Patient Can Walk for PA

• Low / no-impact activities – Aqua classes, water walking – Recumbent bikes / elliptical trainers – Chair aerobics – Walking • Balance training • Strength training • Consider referral to certified trainers, physiologists, or physical therapists

Advance intensity and impact based on ABILITY! 193

Low Risk, High Benefit Physical Activity Tools

• Logs, Journals, and Smartphone Apps – Self Monitoring is associated with SUCCESSFUL maintenance • Tracking devices: – Wearable vs. Smart phone • Exercise Tubing/Hand weights • Physiology ball • Stretching Strap

194 Accelerometers/Pedometers Get You Moving

Individuals who track their movement: • Walk 2,491 extra steps per day • That’s 150 extra kcal/day burned walking • 15 pounds/year possibly lost Counseling patients to exercise without a specific intervention is ineffective.

Get them tracking!

195 Rowland. Journal of Fam Practice. 2008

Pedometer/Accelerometer Intervention

• Advice alone will not kick-start exercise • This office-based intervention does change behavior • Set a step goal and keep a step diary – Goal 10,000 steps per day – Average person does 3,000 to 4,000 steps/day – Every 2,000 steps = 1 mile (100 cal) – Start by measuring the patient’s baseline steps/day – Increase steps weekly as tolerated

196 Physical Activity Summary

1. Know the Physical Activity Guidelines and share them with your patients. They can’t hit the mark if they don’t know the goal. SUCCESSFUL WEIGHT MAINTENANCE DEPENDS ON IT! 2. Work together to determine the physical activity that meets their personal and individualized needs. 3. Identify appropriate resources. 4. Self-track progress and REVIEW together at follow-up visits.

197

Polling Question

According to the 2018 physical activity guidelines, the amount of exercise recommended for weight maintenance is:

• 150 minutes per week of moderate physical activity including 2-3 days with strength training • 150 minutes per week of moderate physical activity including daily strength training • 150-300 minutes or more of moderate physical activity including 2-3 days with strength training • 150-300 minutes or more of moderate physical activity including daily strength training

198 Polling Question

Which statement best describes successful weight loss maintainers who participate in the NWCR (National Weight Control Registry®)?

• Rarely eat breakfast • 75 % weigh at least once weekly • 90 % watch TV less than 1 hour per week • 90 % exercised more than 15 hours per week

201

Further Educational Resources

1. Exercise Physiology: Karli Burridge, PA-C, MMS, FOMA 2. Evaluating Risk for Physical Activity Prescriptions: Karli Burridge, PA-C, MMS, FOMA 3. The New US Physical Activity Guidelines: Upgrading your Understanding of Exercise Prescriptions: Deborah Bade Horn, DO, MPH, FOMA 4. Point-Counterpoint Discussion: How Much Physical Activity do Patients Really Need? Karli Burridge, PA-C, MMS, FOMA and Stephen Phinney, MD, PhD 5. Download the Obesity Algorithm at ObesityAlgorithm.org

www.OMAcademy.org

204 Fundamentals of Obesity Treatment:

Behavior

97 Objectives

• Recognize how behavior influences appetite • Understand the role of neuromodulators in cognitive choice • Learn tools for behavioral therapy use to treat the patient with obesity • Recognize the impact of sleep disturbances on weight • Identify some common eating disorders

207

Nutritional therapy focuses on what you eat

Behavioral Therapy Focuses on why you eat

208 Why Behavior Therapy Is Important…Why We Eat The Way We Do

• Strong biologic forces that resist weight loss • Weak biologic forces that resist weight gain • Behavioral influences can overcome resistance to weight gain – Sensory input – Chronic stress – Mood disorders – Food cravings – Maladaptive food behaviors – eating disorders

209

Why Behavior Therapy Is Important…Why We Eat The Way We Do

• Behavioral influences – Chronic stress  limbic system  emotional eating – Chronic stress  hypothalamus  inflammation and loss of appetite regulation – Chronic stress  reduced self-regulation of food intake – Results in enhanced desire for hyperpalatable foods

210 Why It Matters: The Mesolimbic System

Reward Mood

211

Neurotransmitter: Dopamine

Dopamine • Modulates rewarding properties of food (reward circuit) • External stimuli (desirable food)  cognitive & emotional response  reward-based decision making – Cravings – Hedonic response – eating pleasure – Reward response is greater in those with obesity and in a calorie-restricted state

Gadde, K. M., Martin, C. K., Berthoud, H. R., & Heymsfield, S. B. (2018). Obesity: pathophysiology and management. Journal of the American College of , 71(1), 69‐84. 212 Neurotransmitter: Serotonin

Serotonin • Influences emotional eating behaviors • Emotional states can result in uncontrolled eating • Eating desire in the presence of food • Eating impulsivity • Disinhibition (eating past the point of fullness) • Emotional eating • Binge eating

213 Wurtman, R. Brain Serotonin, Carbohydrate-Craving,, Obesity and Depression. November 1995.

• Useful Behavioral Tools in Obesity Medicine – Stages of Change –5 A’s – Motivational Interviewing – Cognitive Behavioral Therapy

214 Goals of Behavioral Therapy • Patients need to: – Express a desire for changing behaviors (5A’s, Stages of Change) – Recognize unhealthy behaviors (5A’s, MI) – Identify triggers or driving forces behind behaviors (MI, CBT) – Recognize ambivalence toward change (MI) – Plan strategies to overcome stimulus to engage in unhealthy behavior (5A’s, CBT) – Set goals for change (5A’s, CBT) – Be held accountable for changing behavior (5A’s, CBT) – Develop strategies to overcome ineffective responses to relapses (5A’s, CBT)

215

Goals of Behavioral Therapy

• Providers need to: – Recognize a patient’s unhealthy behaviors (5A’s) – Recognize a patient’s willingness/motivation to change (Stages of change, MI) – Recognize a patient’s resistance to change (MI) – Help develop strategies for change (CBT) – Help set goals for change (5A’s, CBT) – Hold patient accountable for change (5A’s, CBT) – Help patient overcome relapses and resume change strategy (5A’s, CBT)

216 Stages of Change

Pre-contemplation Progress Unawareness of the problem

Contemplation Thinking of change in the next 6 months

Preparation Making plans to change now

Action Implementation of change

Maintenance Relapse Continuation of favorable behavior Restart of unfavorable behavior

Ries AV, Blackman LT, Page RA, Gizlice Z, Benedict S, Barnes K, Kelsey K, Carter-Edwards L: Goal setting for health behavior change: evidence from an obesity intervention for rural low-income women. Rural Remote Health 2014 14:2682.

217

Stages of Weight Loss and Stages of Change

• Assess readiness • “Staging” an individual’s readiness to match the targeted behavior for workable, personalized treatment plan and its intended success – meet them where the are – Patients in precontemplation are not ready for action • Help patients turn thoughts and intentions into actions and behaviors

Sutton, K, et al. Assessing Dietary and Exercise Stage of Change to Optimize Weight Loss Interventions. Obesity, Volume 11, Issue 5, May 2003: 641-652.

218 The 5 A’s of Obesity Management

• Ask for permission to discuss body weight. Ask • Explore readiness for change.

• Assess BMI, waist circumference, and obesity stage. Assess • Explore drivers and complications of excess weight.

• Advise the patient about the health risks of obesity, the benefits of modest Advise weight loss (i.e., 5-10 percent), the need for long-term strategy, and treatment options.

• Agree on realistic weight-loss expectations, targets, behavioral changes, Agree and specific details of the treatment plan.

• Assist in identifying and addressing barriers; provide resources; assist in Arrange/Assist finding and consulting with appropriate providers; arrange regular follow up.

219 Obesity Algorithm®. ©2017-2018 Obesity Medicine Association.

Motivational Interviewing

What is Motivational Interviewing (MI)?

Motivational interviewing is a collaborative conversation style to strengthen a person’s own motivation and commitment to change.

Miller and Rollnick Motivational Interviewing: Helping People Change 220 3 ed, 2013 Four Processes for MI

• Engaging • Focusing • Evoking • Planning

221

Four Processes for MI

• Engaging – Establishing a mutually trusting and respectful relationship – Express empathy

222 Four Processes for MI

• Engaging – Avoid disengaging • Assessing – asking a bunch of questions • Telling how to fix the problem • Power differential – exerting your role as an authority figure • Labeling – fat, obese (avoid stigma and bias) • Avoid the “righting reflex”

223

Four Processes for MI

• Focusing – Ongoing process of seeing and maintaining direction – Agenda setting – Interview goals – Setting clear objectives

224 Four Processes for MI

• Evoking – Eliciting the patient’s own motivation for change • Open-ended questions – “What have you tried in the past?” • Affirmations – “So you have had success with…” • Reflections – “It sounds like you are ready to get started.”

225

Four Processes for MI

• Evoking – Change talk vs Sustain talk – Change talk – patient’s speech that favors movement in the direction of change – Identifies a patient’s motivation for change – Sustain talk – patient’s speech that favors maintaining status quo

226 Readiness Scale

• On a scale of 1 to 10, how ready are you to make begin an obesity treatment program? – Ask why not a lower number? • Elicits change talk • Avoids sustain talk • Can also ask: – How important is it for you to…. – How confident are you in your ability to… – How committed are you to…

227

Motivational Interviewing

• Righting Reflex – Avoid trying to correct a patient’s thinking about a condition • “I look at food and I gain weight” • “I know I am heavy, but it’s not affecting my health” • “Whenever I lose weight, I always gain it back” • “Nothing ever works for me”

228 Four Processes for MI

• Planning – Developing a specific plan that the client agrees to and is willing to implement – SMART Goals • Specific – Lose 30 lbs., eliminate sugar sweetened beverages, walk 30 minutes daily. • Measurable – quantify and track the change • Achievable - realistic • Relevant – important to the patient • Timed – specific timeline for change

229

Spirit of MI

– Compassion - Provide support that keeps the patients best interests in mind – Acceptance - Recognize that every person has the right to make their own decisions even if it is a bad one – Partnership - Work together, collaborating on a strategy for change • Avoid playing the expert role…“do what I say” – Evocation - The best ideas come from the patient

230 Cognitive Behavioral Therapy

231

Cognitive Behavioral Therapy

Thoughts

CBT

Behaviors Feelings

232 Behavior Change

Changing attitudes does not change behavior as much as changing behavior changes attitudes - Small actions lead to changes in attitude - Small successes build desire to achieve greater success - Telling patients why they should lose weight is not as powerful motivating force as the effects of patients making changes and losing weight

233

Cognitive Behavioral Therapy

234 Components of Behavioral Therapy (CBT)

1. Self-monitoring – Daily records of food intake, physical activity, and weight 2. Stimulus control – Avoidance; reduce triggers that prompt eating 3. Problem solving – Define the problem, brainstorm solutions, implement strategy 4. Goal setting – “Keep it simple; make 3 changes to diet/exercise for the next 3 weeks” 5. Contingency management – Develop recovery methods when and if there is a relapse

235 Wadden TA. Med Clin North Am. 2000. Wisotsky W, Swencionis C. Cognitive-behavioral approaches in the management of obesity. Adolesc Med 2003: 14:37-49

Components of Behavioral Therapy (CBT)

6. Enlisting social support – Recruiting family and friends is important for modifying lifestyle behaviors 7. Relapse prevention training – Expect setbacks; be prepared, view as temporary 8. Stress management – Decrease the negative impact of stress on positive behavior patterns

236 Wadden TA. Med Clin North Am. 2000. Wisotsky W, Swencionis C. Cognitive-behavioral approaches in the management of obesity. Adolesc Med 2003: 14:37-49 Cognitive Therapy Techniques

• Reframing - reinterpretation of an event • Rehearsal or role playing – example, if offered food, respond with, “I just ate, but would love a cup of…” • Utilizing alternative choices – examples, get smaller portions of meals, take ½ meal home when dining out • Visualize it – have patients see the choices before they are in the situation to ease the burden

237 Gade, H. Effectiveness of a CBT for Dysfunctional Eating among patients admitted for bariatric surgery: a RCT. Journal of Obesity Vo 2014.

Examples of Common Behavioral Interventions

• Diet and physical activity logs • Nutrient logs (phone apps, etc.) • Fitness monitoring (step counters and accelerometers) • Web-based • Cell phone applications • Commercial weight loss programs and support groups

238 National Weight Control Registry (NWCR)

Long-term tracking >10,000 enrollees (80% female, 20% male) The average maintainable weight lost was 66 lbs. for 5.5 years

KEY BEHAVIORAL CHANGES: • 98% participants reported modified food intake (changed eating HABITS) • 94% participants increased physical activity (changed sedentary LIFESTYLE) • 78% eat breakfast daily, 75% self-weigh weekly, 62% have <10 hours screen time/week, 90% exercise 60 minutes/day

239 www.nwcr.ws/research.

National Weight Control Registry

• Common components of successful NWCR participants: – Self-monitoring – Goal setting – Problem-solving skills – Stimulus control – Behavior contracting

240 www.nwcr.ws/research Do Behavior Interventions Work? • Diabetes Prevention Program (DPP) – 27 centers in the US with over 3200 participants – Clearly defined weight loss (7% IBW) and physical activity goals – Lifestyle coaches with frequent contact to assist in achieving and maintaining goals – Individualize treatment strategies – Support groups

241 The Diabetes Prevention Program, Diabetes Care. 2002 Dec; 25(12):2165-2171.

Do Behavioral Interventions Work? • DPP Results – Improved self-monitoring skills – Dietary changes - ↓ total fat / ↓ total calories – Engaged in supervised PA ≥2x / week – Resulted in a 7% average weight loss – ↓ progression to T2DM by 58%

242 The Diabetes Prevention Program, Diabetes Care. 2002 Dec; 25(12):2165-2171. Do Behavior Interventions Work? • Look AHEAD (Action for Health in Diabetes) – RTC 5000 participants – intensive lifestyle interventions (ILI) – 5 years study – Goal - 10% weight loss and then prevention of weight gain – Behavior interventions • Portion control • Lifestyle activities methods/strategies – NEAT – non-exercise activities – Exercise activities •.

243 The Look AHEAD Study: A Description of the Lifestyle Intervention and the Evidence Supporting It. Obesity. 2006 may; 14(5):737-752.

Do Behavior Interventions Work?

• Look AHEAD Outcomes – Greater weight loss in ILI group 6.0% vs 3.5% – Greater reduction in A1c and most CVD risk factors

Look AHEAD Research Group. (2013). Cardiovascular effects of intensive lifestyle intervention in type 2 244 diabetes. New England journal of medicine, 369(2), 145‐154. . Do Behavior Interventions Work?

• Look AHEAD Take home points: – Behavior modifications work – Frequent visits needed (monthly if feasible) – Both group and individualized support sessions effective – Self-monitoring and awareness are key to success • Learning to reverse small weight regains • Critical skill for maintaining long-term weight loss

Look AHEAD Research Group. (2013). Cardiovascular effects of intensive lifestyle intervention in type 2 245 diabetes. New England journal of medicine, 369(2), 145‐154. .

Co-Morbidities of Obesity and the Influence of Behaviors

• Sleep Disorders – Shift work • Eating Disorders – Binge-eating disorder – Night-eating syndrome

246 Sleep, Obesity, and Behaviors

• Epidemiologic link between sleep duration and adverse health outcomes and obesity • associated with: • Decreased leptin, increased ghrelin, leading to increased appetite and decreased energy expenditure • Hormonal changes affect mood and behaviors • Result in poor food choices

Cappuccio FP, Meta-analysis of short sleep duration and obesity in children and adults. Sleep 2008; 31:619-26 rd 247 Bray and Bouchard, Handbook of Obesity, 3 edition

Sleep Disturbance - Major Causes

• Poor sleep habits and sleep hygiene • Medications and substances that interfere with restorative sleep • Sleep disorders – Obstructive and other sleep apnea syndromes – Obesity-hypoventilation syndrome – Shift work sleep disorder – Restless leg syndrome and other sleep disturbances – Sleep related eating disorder (considered a sleep disorder)

248 Shift Work Sleep Disorder

• Diagnostic criteria – Chronic / recurrent pattern of sleep-wake disruption caused by alteration of normal circadian rhythms – Sleep-wake disturbance with or excessive sleepiness – Associated with impairment and behaviors that affect poor food choices

Bray and Bouchard, 3rd Ed pg 398 249 Morgenthaler T, et al, Practice Parameters for the Clinical Evaluation and Treatment of Circadian Rhythm Sleep Disorders, AASM

Shift Work Sleep Disorder

• Treatment • Planned sleep schedule • Manage light exposures • Bright light exposure at work • Limited light exposure before bedtime • prior to sleep time • Modafinil during awake time • Avoid stimulants near bedtime, ex. caffeine

250 Eating Disorders

• Consider: other conditions may co-exist making diagnosis a challenge – MDD, GAD, OCD, PTSD, substance abuse • Food is rarely a source of pleasure for these patients – Love/hate, friend/enemy, impulse/shame • Breeding ground for anger and resentment towards self – Passive-aggressive anger toward others • Oftentimes a person of loneliness – Ex: “People in their lives untrustworthy, but FOOD is reliable”

251 Klump KL. Psychiatr Clin North Am. 2001.

Eating Disorders: Binge-eating Disorder

Diagnosis: • Frequent episodes of consuming large amounts of food more than once per week for at least three months – No self-induced vomiting (purging) – No extra exercising – Feelings of lack of self control, shame, and guilt • Occurs in 2-3 percent of U.S. adults, may occur up to 50% in those with obesity (most common eating disorder) Treatment: • Cognitive behavior therapy • Lisdexamfetamine dimesylate: FDA indication for treatment of binge-eating disorder

252 Obesity Algorithm®. ©2017-2018 Obesity Medicine Association. Eating Disorders and Obesity: Night-eating Syndrome

Diagnosis: • At least 25 percent of daily food consumption (often greater than 50 percent) consumed after evening meal • Recurrent awakenings from sleep that require eating to go back to sleep, often involving carbohydrate-rich snacks • Little interest in breakfast • May occur in as much as 5 percent of the U.S. population Treatment: • CBT focusing on nutritional timing and content

253 Obesity Algorithm®. ©2017-2018 Obesity Medicine Association.

Behavior Clinical Pearls

1. Behaviors are affected by brain neurotransmitters, specifically dopamine and serotonin, and beyond an individuals willpower. 2. Recognizing that positive behavioral change can be achieved and are integral to clinically significant weight management. 3. Stages of Change, 5A’s, motivational interviewing, and cognitive behavioral therapy are all tools for behavioral change that can be used to treat the patient with obesity. 4. Sleep disorders, eating disorders and mood disorders can contribute to dysfunctional eating behaviors

254 Polling Question

A patient with diabetes and class II obesity is expressing ambivalence toward starting an obesity treatment plan. Which of the following would be an example of a motivational interviewing technique?

• “I realize that you have lost weight and gained it back in the past, but it is important for your diabetes to lose weight now.” • “Setting a 20-pound weight loss goal over the next 3 months is a reasonable place to start.” • “What keeps you from starting the program now?” • “You should start a low-carbohydrate diet, it will work best for diabetes.” • “Your obesity is causing your diabetes.”

255

Polling Question

The parents of an 8-year-old girl are asking for recommendations to help their child lose weight. They usually keep soda in the house which the child drinks regularly throughout the day. Recommending that they keep flavored seltzer water in the home rather than soda is an example of:

• Contingency management • Goal setting • Self-monitoring • Stimulus control • Stress management

258 Further Educational Resources

1. Motivational Interviewing: What, Why and How: Colleen Fairbanks, PhD 2. Managing Overeating: The Key Intersection between Psychology and Neuroscience: David A. Macklin, MD, CCFP and Sandy Van, MD, CCFP, Dipl. ABOM 3. Treatment of Eating Disorders in the Era of Healthy at Every Size: Nicole Garber, MD 4. The Relationship between Obesity and Sleep: Craig Primack, MD, FACP, FAAP, FOMA, Dipl. ABOM 5. Download the Obesity Algorithm at ObesityAlgorithm.org

www.OMAcademy.org

261 Fundamentals of Obesity Treatment:

Medication

127 Always check with your local and state medical/ boards before prescribing anti-obesity medications.

Local and state laws and regulations may vary.

264

Why Use Medication with Obesity Treatment?

• Weight loss provokes a complex set of neuroendocrine physiologic adaptations that become more intense with greater weight loss. These work to slow, then eventually halt weight loss, and eventually may induce weight gain.

• Patients who have lost weight find it very difficult to resist neuroendocrine physiology with diet and behavior modification alone. • Anti-obesity Medication help offset the physiologic adaptations that resist weight loss and promote weight regain

Greenway, International Journal of Obesity (2015) 39, 1188–1196; doi:10.1038/ijo.2015.59; published online 26 May 2015 265 Anti-obesity Medications

Adjunct to nutritional, physical activity, and behavioral for patients with BMI ≥ 30 or BMI ≥ 27 with co-morbidities Objectives: • Treat disease ‒ Adiposopathy or sick fat disease (SFD) ‒ Fat mass disease (FMD) • Facilitate management of eating behavior • Facilitate weight loss and slow progression of weight gain/regain • Improve the health, quality of life, and body weight of the patient with overweight or obesity • Counter physiologic adaptation to calorie restriction

5-10 percent weight loss may improve both metabolic and fat mass disease

266 Reference/s: [239] Obesity Algorithm®. ©2016-2017 Obesity Medicine Association.

Potential Mechanisms of Action • Decrease appetite/cravings Obesity Drugs alter physiology, not only behaviors • Decrease leptin resistance • Increased energy expenditure • Increase adherence by mitigating biological or genetic factors

York DA. Nutrition.2000;16:967-75 267Adapted from Ioannides-Demos LL, et al. J Obes 2011 Liu et al. Cell 2015;161, 999–1011. Why Use Medication with Obesity Treatment?

Anti-obesity medications and metabolic (sleeve gastrectomy, RNY gastric bypass, duodenal switch) change the physiology of body weight regulation and offer additional therapeutic options to attain long-term success.

268 Bray, Obesity 2013;21:893.

Guidelines/Algorithm

• Plethora of guidelines

– most are BMI-centric recommending no anti-obesity drug until BMI thresholds reached • State medical boards may • OMA Obesity Algorithm® censure off-schedule anti-obesity medication use – a patient-centric, complication- centric model • Documentation is key

• Guideline resources - AHA/ACC/TOS – DOI: 10.1016/j.jacc.2013.11.004 - AACE/ACE – doi.org/10.4158/EP161365.GL

269 - Endo - doi.org/10.1210/jc.2014-3415 Guidelines/Algorithm – Limitations

• Obesity “defined” as BMI ≥ 30

• Overweight BMI ≥ 25 Kg/m2

• Example: average woman must gain ~55 pounds of excess fat to reach BMI 30

• Example: average woman must gain ~35 pounds of excess fat to reach BMI 27

• Systemic inflammation begins early - long before weight gain of 35 pounds

• Adiposopathy frequently begins prior to reaching a BMI of 25

270

Good Practices

Document:

– Initial drug prescription rationale

– Vital signs, appearance, mood, every visit

– Beneficial and adverse effects every visit

– Rationale for any prescription changes Also:

– Participate in obesity medicine education

– Consult with obesity medicine specialists

271 Why Use Medication with Obesity Treatment?

Medication is an integral treatment modality for all patients and should be considered at every stage of the disease and at every visit.

272

Why Use Medication with Obesity Treatment?

273 Why Use Medication with Obesity Treatment?

• Think of anti-obesity medications as agents that can induce improvement or retard progression of obesity-associated illnesses through weight loss, weight- loss maintenance, or retardation of progressive weight gain. • Thus these drugs can assist in managing the 195 illnesses that are induced or aggravated by excess adiposity. • Typically, but not always, weight loss induces the other benefits so we think of “responders” as those who have at least 5% weight loss in 3 months. • Although the drug manufacturers and FDA regulations recommend stopping medication if weight loss is less than 5% after 3 months, it is also important to look for other benefits before discontinuing anti-obesity medications (example: patient is no longer needing insulin to control diabetes).

Yuen, M. S., Lui, D. T., & Kaplan, L. M. (2016). A systematic review and evaluation of 274 current evidence reveals 195 obesity-associated disorders (OBAD). Obesity Week.

Treatment Combinations

• Combine diet, physical activity and other behavioral changes with long-term pharmacotherapy for optimal success. • As is the case in most chronic diseases, optimum management may require more than one medication. Combinations often work better than a single medication. • Examples to be discussed include the combinations of naltrexone and bupropion, and also phentermine and topiramate.

275 Long-term Prescription Use in Obesity Treatment

Diet & exercise

Diet, exercise, & phentermine

Year

276 Hendricks, Obesity. 2011;19:2351.

Long Term Use of Medication: Greater Success and Diabetes Prevention

Three years of liraglutide versus placebo for type 2 diabetes risk reduction and weight management in individuals with prediabetes: A randomised, double-blind trial.

le Roux C.W., Astrup A., Fujioka K., Greenway F., Lau D.C.W., Van Gaal L., Ortiz R.V.,Simon J.A. (2017) The Lancet, 389 (10077) ,pp. 1399-1409.

277 Combine Medication with Behavior Modification

Additive effects of behavior modification in addition to medication therapy shown here using Contrave® (naltrexone/bupropion)

Weight Loss With Naltrexone SR/Bupropion SR Combination Therapy as an 278 adjunct to Behavior Modification: The COR-BMOD Trial Wadden et al Obesity (Silver Spring). 2011 Jan; 19(1): 110–120

Combine Diet, Medication, and Behavior Modification

0 Additive effects of Medication alone behavior and meal 5 replacement Weight Medication and behavior therapy with 10 Loss modification pharmacotherapy (%) for obesity 15 Medication, behavior modification, and meal replacements

20

2 4 6 8 10 12 Time/Months

279 Wadden, Arch Int Med. 2001;161:218 Barriers to the Use of Medication in Treating Obesity

• Perception that obesity is a disorder of willpower or lack of self- control and can be simply treated by limiting calories and increasing physical activity • The failure to perceive obesity as a chronic incurable disease that requires lifelong intervention has resulted in the perception that weight regain after termination of treatment is due to failure of the medication • Regulatory rigidity that limits medication usage and duration of therapy • Inadequate funding for clinical obesity research • Lack of coverage for anti-obesity medications by health plans

280 Bray GA, Ann Intern Med. 1993;119:707

Anti-Obesity Medication Benefits

• Anti-obesity medications counteract hedonic and/or physiologic drives that result in unhealthy eating behaviors and obesity related disease. Benefits may include:

• Improvement or remission of obesity-related diseases

• Weight loss

• Maintenance of weight loss

• Diminution or disappearance of cravings

• Changes in eating behaviors

– Obsessive eating

281 – Improved eating control and diet adherence The Medications: FDA Approved for Long-term Usage

Orlistat Lorcaserin Phentermine/ Topiramate ER Trade Names: Xenical (1999) Trade Name: BELVIQ® Trade name: Qsymia® or alli® (2007) (2012) (2012)

Naltrexone/ Liraglutide Bupropion ER Trade name: Contrave® Trade Name: Saxenda® (2014) (2014)

282

Orlistat (alli® or Xenical®)

• Obesity Indication • Chronic Weight Management

• OTC – 60 mg TID w/ meals Dosage • Rx – 120 mg TID w/ meals

Mechanism of action • Gastric lipase inhibitor (MOA) • Pancreatic lipase inhibitor

• Pregnancy Contraindications • Chronic malabsorption • Cholestasis • Diarrhea, oily stools Common adverse drug • Fecal incontinence reactions (ADR) • Fat soluble vitamin deficiency • 1-year 3.9% 2-year 2.3% Weight loss • ≥5% - 21% ≥10% - 12%

**May be used in patients as young as 12 years of age FDA approved 1999 (alli® approved for OTC in 2007) 283 Prescribing Information of Xenical http://www.roche-australia.com/home/products/pharmaceuticals/xenical.html Lorcaserin (BELVIQ®)

•BMI ≥ 30 kg/m2 Indication •BMI ≥ 27 kg/m2 + 1 ARD

• 10 mg BID Dosage • XR 20 mg daily

Mechanism of action • Selective serotonin 5HT2c agonist (MOA) • Activates POMC neurons - ↑satiety

Contraindications • Pregnancy • Hypersensitivity • Potential for serotonin syndrome Common adverse drug • Headache, dizziness, fatigue reactions (ADR) • Nausea, dry mouth, constipation • Hypoglycemia, back pain, cough • 5.8 Kg (12.8 lbs.) Weight Loss • ≥5% WL - 47%; Placebo 22% • ≥10% WL - 22%; Placebo 8.7%

**DEA Schedule IV FDA approved 2012 284 Prescribing Information BELVIQ® (www.Belviq.com)

Phentermine/Topiramate ER (Qsymia®)

•BMI ≥ 30 kg/m2 Indication •BMI ≥ 27 kg/m2 + 1 ARD • 3.75/23 mg x 14d → ↑7.5/46mg Dosage • If <3%WL at 12 weeks then increase • 11.25/69mg x 14d → ↑15/92mg Mechanism of • Phentermine – sympathomimetic action (MOA) • Topiramate - ↑GABA / ↓carb. anhy. • Pregnancy Contraindications • Glaucoma, hyperthyroidism • Uncontrolled HTN

Common adverse • Headache, dizziness, fatigue drug reactions • Nausea, dry mouth, constipation (ADR) • Hypoglycemia, back pain, cough

Weight Loss • ≥5% WL - 67%; Placebo 17% • ≥10% WL - 47%; Placebo 7%

**DEA Schedule IV 285 FDA approved 2012 Prescribing Information of Qsymia® (www.qsymia.com) Naltrexone/Bupropion HCl ER (CONTRAVE®)

•BMI ≥ 30 kg/m2 Indication •BMI ≥ 27 kg/m2 + 1 ARD • 8mg naltrexone/90mg buproprion Dosage • Titrate 1 qAM x 7d; then 1 BID x 7d • Then 2 qAM, 1qPM x 7d; then 2 BID Mechanism of • Naltrexone – opioid antagonist action (MOA) • Bupropion -↓NE & dopamine uptake • Pregnancy Contraindications • Uncontrolled HTN, Seizure d/o • Opioid use, eating disorder, MAOI use

Common adverse • Headache, dizziness, insomnia drug reactions • Nausea, dry mouth, (ADR) • Constipation, diarrhea

• WL 5.4 & 8.1%; Placebo 1.3% & 4.9% Weight Loss • ≥5% WL – 42&57%; Placebo 17&43% • ≥10% WL – 21&35%; Placebo 7&21%

286 FDA approved 2014 Prescribing Information CONTRAVE ® (www.contrave.com)

Liraglutide (Saxenda®)

•BMI ≥ 30 kg/m2 Indication •BMI ≥ 27 kg/m2 + 1 ARD • Titrate: 0.6mg SC daily x 7d, Dosage • then 1.2mg x 7d, then 1.8mg x 7d, • then 2.4mg x 7d, then 3.0mg x 7d Mechanism of • GLP-1 agonist action (MOA) • Central satiety, ↓gastric emptying • Pregnancy Contraindications • Personal or FH medullary thyroid CA • MEN type 2, caution h/o pancreatitis

Common adverse • N/V, diarrhea, constipation drug reactions • Headache, dizziness, fatigue (ADR) • Hypoglycemia, abdominal pain

• WL 9.2% vs 3.5% Placebo Weight Loss • ≥5% WL – 62%; Placebo 34% • ≥10% - 34%; Placebo 15%

FDA approved 2014 287 Prescribing Information Saxenda ® (www.Saxenda.com) Cardiovascular Outcomes with New Anti-obesity Medications

Medication LDL TG HDL A1c SBP

Phentermine/Topiramate ↓↓↑↓ ↓ CR

Lorcaserin ↓↓ ↓ _ ↓/--

Naltrexone SR/Bupropion _ ↓↑↓ ↑ SR

Liraglutide 3.0mg ↓↓↑↓↓

Journal of the American College of Cardiology, Volume 68, Issue 8, August 2016 Cardiovascular Effects of the New Weight Loss Agents 288 Matthew H. Vorsanger et al

The Medications: FDA Approved for Short-term Usage

Sympathomimetics - Phentermine - Diethylpropion - Phendimetrazine Benzphetamine

289 Sympathomimetics (SMPs)

• Drug class: phenethylamines – includes amphetamine, methamphetamine, phentermine, diethylpropion, epinephrine, dopamine, and many others. • Phentermine is not an “amphetamine.”

• Phentermine FDA approval in 1959 during a U.S. epidemic of amphetamine addiction. Presumption then – all SMPs shared same adverse effects including addiction potential. • All obesity drugs reapproved in 1970s for “short-term use” only due to continuing concerns of addiction despite the fact that addiction had occurred only with amphetamine. • Prescribing these medications for more than three months is considered “off label.” There is abundant experience and literature demonstrating the safety and efficacy of these drugs well beyond twelve weeks.

Apovian CM, Aronne LJ, Bessesen DH, et al. Pharmacological management of obesity: an Endocrine Society clinical practice guideline. 290 J Clin Endocrinol Metab. 2015.100(2):342–362.

Phentermine

•BMI ≥ 30 kg/m2 Indication •BMI ≥ 27 kg/m2 + 1 ARD

• 8 mg ½ to 1 qd to tid Dosage • ER 15mg, 30mg, 37.5mg scored tab

Mechanism of • Sympathomimetic – centrally acting action (MOA) •NE ↑ release/↓ uptake → ↓appetite

Contraindications • Pregnancy, Glaucoma, CVD • Uncontrolled HTN, hyperthyroidism

Common adverse • Headache, dizziness, fatigue drug reactions • Nausea, dry mouth, constipation (ADR) • Hypoglycemia, back pain, cough

Weight Loss • Limited data

**DEA Schedule IV Controlled Substance 291 Prescribing Information Saxenda ® (www.Saxenda.com) Phentermine: Common Misperceptions on Adverse Effects

Addiction – Addiction potential in clinical setting: no evidence – Withdrawal: no evidence of amphetamine-like withdrawal

Adverse cardiovascular effects – No established relationship related to cardiac valvulopathy or pulmonary hypertension

Rothman AJT 2010, Hendricks Obes 2011 292

Diethylpropion

•BMI ≥ 30 kg/m2 Indication •BMI ≥ 27 kg/m2 + 1 ARD

• 25mg tid Dosage • SR 75mg

Mechanism of • Sympathomimetic – centrally acting action (MOA) •NE ↑ release/↓ uptake → ↓appetite

Contraindications • Pregnancy, Glaucoma, CVD • Uncontrolled HTN, hyperthyroidism

Common adverse • Headache, dizziness, fatigue drug reactions • Nausea, dry mouth, constipation (ADR) • Hypoglycemia, back pain, cough

Weight Loss • Limited data

**DEA Schedule IV Controlled Substance 293 Cercato, Int J Obes. 2009;33:857 Phendimetrazine

•BMI ≥ 30 kg/m2 Indication •BMI ≥ 27 kg/m2 + 1 ARD

• 35mg capsule tid Dosage • SR 105mg capsule daily

Mechanism of • Sympathomimetic – centrally acting action (MOA) •NE ↑ release/↓ uptake → ↓appetite

Contraindications • Pregnancy, Glaucoma, unstable CVD • Uncontrolled HTN, hyperthyroidism

Common adverse • Headache, dizziness, fatigue drug reactions • Nausea, dry mouth, constipation (ADR) • Hypoglycemia, back pain, cough

Weight Loss • Limited data

**DEA Schedule III Controlled Substance Cass, Can Med Assoc J. 1961;84:1114 294 Le Riche, Can Med Assoc J. 1962;87:29

Benzphetamine

•BMI ≥ 30 kg/m2 Indication •BMI ≥ 27 kg/m2 + 1 ARD • 25-50 mg one to three times daily. Dosage • Some patients may respond to a single mid-morning or mid-afternoon dose • Sympathomimetic – centrally acting Mechanism of action •NE ↑ release/↓ uptake → ↓appetite (MOA) • Active metabolites include d-amphetamine and d-methamphetamine*

• Pregnancy, Glaucoma, unstable CVD Contraindications • Uncontrolled HTN, hyperthyroidism

Common adverse drug • Dizziness, elevated BP, palpitations reactions (ADR) • Confusion, insomnia, tremor • Diaphoresis, headache, dry mouth

Weight Loss • Limited data

296 **DEA Schedule III Controlled Substance *Banks, ML, et al Pharmacol Biochem Behav. 2017 The Medications: Off-label Usage for Weight Loss

Sympathomimetics Metformin Topiramate Greater than 12 weeks

Generic Combination Generic Combination Naltrexone / Phentermine / Bupropion Topiramate

297

Metformin

2% in type 2 diabetes and insulin resistance or impaired Weight loss fasting glucose 500 to 2000 mg/day Dose Daily dosing with XR formulation or BID dosing with immediate release tablets Activates AMPK; decreases hepatic glucose production, Mechanism of action increases muscle glucose uptake Nausea, abdominal cramping, other GI ASEs Adverse side effects Avoid in patients with renal dysfunction (GFR < 30%) Metformin is FDA approved only for the treatment of type 2 diabetes in adults and children May be used used as adjunctive in combination with other anti-obesity medications in patients with insulin resistance. If intolerance to GI symptoms consider using metformin ER/XR.

298 Diabetes Prevention Program (DPP). 2012 G Zhou, J Clin Invest. 2001 Tejas P. Desai. Metformin in Patients With CKD: A New Analysis - Medscape - Mar 23, 2015. Metformin

Prevention of type 2 diabetes (JAMA. 2017;317(11):1171. doi:10.1001/jama.2016.17844)

May help improve adiposopathic disorders: • Insulin resistance • Polycystic ovarian syndrome • Fatty liver • Cardiovascular disease (especially when compared to sulfonylurea)

May help treat complications of other concurrent drug treatments: • Antipsychotic-related weight gain • Human immunodeficiency virus (HIV) protease inhibitor-associated abnormalities (i.e., HIV )

May help reduce the overall cancer rate and help improve the treatment of multiple cancers: • Colon, Ovary, Lung, Breast, Prostate

May enhance effects of gastrointestinal hormones applicable to weight loss (e.g., glucagon-like peptide-1, peptide YY)

May alter gut microbiome

299 Obesity Algorithm®. ©2016-2017 Obesity Medicine Association. Reference/s: [115-118]

Topiramate

• Approved for seizures in 1996 and 2004 for prevention of migraines • Except in Qsymia®, topiramate is not approved for the treatment of obesity. Several studies have shown encouraging results for the use of topiramate (alone or in combination with sympathomimetic amine anorectics) for the treatment of and obesity. • Typical dosing – Epilepsy: 400 mg/day – Migraines: 200 mg/day – Obesity: 25 – 100 mg/day • Start dosage at 25 mg in the pm; increase dose every two weeks by 25 mg (usually bid) based on clinical response, tolerance, or ASE’s

300 Topiramate: Adverse Effects

• Attention difficulty, memory loss

• Fatigue,

• Acute myopia and angle closure glaucoma (rare)

• Kidney stones (rare)

• Paresthesias, dysgeusia

• Depression, anxiety, suicidal ideation

• Increased risk of oral clefts if taken during pregnancy in first trimester. Use extreme caution in child bearing age.

301

Biodegradable Hydrogel Capsule Device

• The FDA has cleared a non-systemic capsule consisting of cross-linked molecules of cellulose and citric acid forming a matrix that rapidly absorbs water in the stomach, forming individual gel pieces. • The gel pieces increase the volume and elasticity of the stomach and increase the feeling of fullness thereby reducing energy intake and leading to weight loss. • Clinical trials: 59 % of subjects achieved ≥ 5% weight loss at 6 months (6.4 % vs 4.4 % placebo) • 26 % achieved ≥ 10 % weight loss

302 Biodegradable Hydrogel Capsule Device

• Indication: For treatment of overweight and obesity: (BMI≥25 kg/m2) used in conjunction with appropriate nutrition and physical activity. There are no restrictions for duration of therapy. The hydrogel capsule is technically a device, not a pharmaceutical agent.

• Precautions: It may interfere with absorption of other medications, especially those taken with meals. Avoid using in patients with esophageal abnormalities, such as webs, stricture, rings, or diverticuli. Use with caution in GERD, ulcers, heartburn or a history of GI surgery complications that may alter motility.

• Adverse effects: GI-related; pain, flatulence, bloating, constipation

• Contraindications: Pregnancy and to ingredients

• Dosing: 3 capsules with water followed by 16 oz of fluid 20-30 minutes prior to each meal 303

Special Situations

• Depression – bupropion (might be first choice in patients with obesity)

• Binge eating disorder –FDA approved for this condition. Lisdexamfetamine (Vyvanse®) initial dosage 30 mg qd in am; titrate in increments of 20 mg weekly; usual maintenance dose is 50-70 mg q AM; topiramate (not FDA approved for binge eating disorder); 25 mg in the PM; titrate up to 100mg BID if needed

• Night eating syndrome – sertraline, citalopram, escitalopram

• Diabetes: best choices for weight loss include metformin, GLP1 agonists, SGLT2 inhibitors, glucosidase inhibitors; weight neutral: DPP4 inhibitors

• Premenstrual carbohydrate cravings: spironolactone in latter half of cycle to second day of menses. Precaution with patients who may become pregnant. Scant medical literature regarding this treatment

304 Anti-Obesity Medication Summary

Drug Side Effects Mechanisms Website/Coupon Support Program Phentermine 15 mg and 30 mg Anxiety, agitation, Decreased appetite and food www.lomaira.com capsules, 37.5mg tablet or palpitations, dry mouth intake Coupon – Yes capsule, and Support - No 8mg dose Lomaira® Phentermine/Topiramate Numbness and tingling, Decreased appetite and food www.qsymia.com (Qsymia®) titrate every 2 weeks word finding issues, intake, taste changes Coupon – Yes 3.75mg/23 mg to7.5mg/46mg to makes carbonated Support – Yes 11.25 mg/69 mg to 15mg/92mg beverages taste bad Mayo diet and online program

Lorcaserin (Belviq®) Fatigue, dry mouth, Decreased appetite and food www.belviq.com 20mg XR daily headache intake, increased feeling of Coupon – Yes 10mg twice a day fullness Support – Yes Online program Naltrexone/bupropion (Contrave®) Nausea, anxiety, Decreased appetite and www.contrave.com 8mg naltrexone/90mg bupropion increase in blood reward Coupon – Yes titrate to 2 tablets BID pressure, do not use Support – No with opioids online Rx program Liraglutide (Saxenda®) Nausea, diarrhea, Reduced food intake, www.saxenda.com 3mg once daily injection constipation, do not use increase feeling of fullness, Coupon – Yes if history of medullary improved blood sugar Support – Yes RD online/telephone coaching 305 thyroid cancer or metabolism pancreatitis

Weight All % patients on % patients on % patients on % patients on loss % medications liraglutide 3mg phentermine/topiramate lorcaserin bupropion/naltrexone combined (Saxenda©) 15/92mg (Qsymia©) (Belviq©)10mg (Contrave©) without after surgery without surgery without surgery BID without surgery surgery

> 5% 54% 62% 67% 47% 42%

> 10% 30% 34% 47% 22% 21%

> 15% 15% 10%

• Patients were twice as likely to lose > 10% with topiramate in the post surgery study • Using medications to optimize pre-surgical weight loss, prevent and/or treat weight regain and produce greater weight loss is important.

306 Medications that May Cause Weight Gain: General Concepts

• The medical literature is often contradictory regarding drug-induced weight gain • Conditions treated may be associated with obesity (e.g. depression, diabetes mellitus, and hypertension) • Changes in lifestyle brought on by these conditions • Not every patient gains weight from medications associated with weight gain • Some people may actually lose weight on these drugs • Some medications associated with weight gain may benefit eating disorders (ex: some SSRI’s)

307

Medications that May Cause Weight Gain: General Concepts

• Most studies are for defined periods of time (usually six weeks to a year) • Most studies focus on the mean or average weight gain or loss, obscuring effect on individual subjects • Studies may be biased as prescribers may utilize drugs less likely to produce weight gain in patients most at risk for weight gain • Some medications are weight neutral or cause weight loss in the short-term followed by weight gain

308 Identify and Manage Concomitant Pharmacotherapy that Might Alter Body Weight

Cardiovascular Medications Diabetes Mellitus Medications

May increase body weight: May increase body weight: • Some beta-blockers • Most insulins ‒ Propranolol • Sulfonylureas ‒ Atenolol • Thiazolidinediones (“-zones”) ‒ Metoprolol • Meglitinides (e.g., nateglinide, repaglinide) ‒ Carvedilol may not increase body weight • Older and/or less lipophilic dihydropyridine (“dipine”) May decrease body weight: calcium channel blockers may increase body weight • Metformin gain due to edema possibly because they are more • Glucagon-like peptide-1 agonists (e.g., “-tides”) vasodilatory), compared to non-dihydropyridines • Sodium glucose co-transporter 2 inhibitors (e.g., “- and lipophilic dihydropyridines. The increased flozins”) edema may exacerbate obesity-related edema (and • Alpha glucosidase inhibitors (e.g., acarbose, miglitol) sleep apnea related peripheral edema), and also • Pramlintide confound body weight as a measure of body fat ‒ Nifedipine Neutral effects on body weight: ‒ Amlodipine • Dipeptidyl peptidase-4 (DPP4) inhibitors (e.g., “-gliptins”)

309 Obesity Algorithm®. ©2020 Obesity Medicine Association.

Identify and Manage Concomitant Pharmacotherapy that Might Alter Body Weight

Hormones Anti-seizure Medications

May increase body weight: May increase body weight: • Glucocorticoids • Carbamazepine • Gabapentin Variable effects on body weight: • Valproate • Progestin contraceptives • Pregabalin ‒ Injectable or implantable progestins may have greatest risk for weight gain May decrease body weight: ‒ May be dependent upon the individual • Topiramate • Testosterone • Zonisamide ‒ May reduce percent body fat and increase lean body mass, especially if used to replace testosterone deficiency in men

310 Obesity Algorithm®. ©2020 Obesity Medicine Association. Identify and Manage Concomitant Pharmacotherapy that Might Alter Body Weight

Antidepressants Variable effects on body weight: May increase body weight (reports on body weight not always consistent): • Some tricyclic antidepressants (secondary amines) • Some tricyclic antidepressants (tertiary amines) ‒ Desipramine ‒ Amitriptyline ‒ Nortriptyline ‒ Doxepin ‒ Protriptyline ‒ Imipramine • Some selective serotonin reuptake inhibitors ‒ Dosulepin ‒ Escitalopram • Some selective serotonin reuptake inhibitors (e.g. ‒ Sertraline paroxetine, citalopram) • Some serotonin and norepinephrine re-uptake • Some selective serotonin and norepinephrine re- inhibitors uptake inhibitors (e.g., venlafaxine) ‒ Desvenlafaxine • Some irreversible monoamine oxidase inhibitors ‒ Duloxetine (e.g., isocarboxazid, phenelzine) • Some irreversible monoamine oxidase inhibitors • Trazodone (i.e., tranylcypromine) • Mirtazapine • Some other serotonergic agents • Brexpiprazole ‒ Vortioxetine May decrease body weight: • Bupropion • Fluoxetine (variable)

311 Obesity Algorithm®. ©2020 Obesity Medicine Association.

Identify and Manage Concomitant Pharmacotherapy that Might Alter Body Weight

Mood Stabilizers Migraine Medications

May increase body weight: May increase body weight: • Gabapentin • Amitriptyline • Divalproex • Gabapentin • Lithium • Paroxetine • Valproate • Valproic acid • Vigabatrin • Some beta-blockers • Cariprazine • Carbamazepine May decrease body weight: • Topiramate Variable/neutral effects on body weight: • Zonisamide • Lamotrigine (sometimes reported to decrease body weight) • Oxcarbazepine

312 Obesity Algorithm®. ©2020 Obesity Medicine Association. Identify and Manage Concomitant Pharmacotherapy that Might Alter Body Weight

Most consistently Antipsychotics Neutral/variable effects increase body weight: on body weight: •Amisulpride • Paliperidone • Clozapine • Quetiapine • Olanzapine • Aripiprazole • Perphenazine • Risperidone • Asenapine • Chloropromazine • Sertindol • Brexpiprazole • Cariprazine • Trifluoperazine • Haloperidol • Iloperidone • Zotepin • Lithium • Loxipine • Lurasidone • Ziprasidone

Hypnotics May increase body weight: May have limited effects on body • Diphenhydramine weight: • Benzodiazepines • Melatonergic hypnotics • Trazodone

313 Obesity Algorithm®. ©2020 Obesity Medicine Association.

Identify and Manage Concomitant Pharmacotherapy that Might Alter Body Weight

Human Immunodeficiency Chemotherapies and Anti-Inflammatory Agents Virus (HIV) Medications May increase body weight: May increase body weight: • Tamoxifen • Some highly active antiretroviral therapies • Cyclophosphamide (HAART) protease inhibitors without HIV • Methotrexate lipodystrophy • 5-fluorouracil • Aromatase inhibitors May decrease body weight: • Corticosteroids • Some highly active antiretroviral therapies (HAART) protease inhibitors with HIV May decrease body weight: lipodystrophy • Apremilast

314 Obesity Algorithm®. ©2020 Obesity Medicine Association. General Principles in Avoiding Medication-associated Weight Gain

• Choose medications that are weight-neutral or less likely to cause weight gain • Strive to use medications that have dual benefits for weight loss and disease management (eg. metformin and/or liraglutide for DM, or topiramate for migraine) • Avoid steroid usage if possible, especially in patients with obesity and obesity- related conditions • If a patient is gaining weight, can’t lose weight, or has inadequate control of an obesity-related disease, review current medications, and if possible, substitute a safe and effective alternative that is less likely to contribute to adiposity-related disease, cause weight gain or interfere with weight loss • Do not discontinue a potential critical medication (eg. an atypical antipsychotic med in a patient with bipolar disorder, or beta blocker in patient with heart failure) unless an effective alternative is available • Always work in conjunction with the prescribing physician 315

Long-term Treatment Strategy

• Most patients will lose 8-15% in the first year and then plateau; some plateau at 6-9 months or earlier • When weight loss slows or plateaus, always reevaluate • Consider changing medicine dose, changing medication, or adding another medication • Ultimately the patient will reach a weight that they cannot get below due to physiology of the weight-reduced state • Most will be disappointed to still be overweight, expectation management is key • The result - many will be so frustrated they will drop out • Dropouts almost always regain the weight loss and more • Repeat weight-loss efforts are almost always less successful

316 Educate Your Patient – Manage Expectations!

Let your patient know in advance:

• Weight loss is often slower than patients expect

• Plateaus are common

• Maintaining weight loss is difficult without an intervention plan

• Medications may be necessary long term or intermittent indefinitely

• There are physiologic drives to restore “lost” energy stores and regain weight

• Obesity is a chronic disease and the disease of obesity is incurable but manageable

• 5-10% weight loss produces health benefits

• Long-term favorable patient health outcomes is the primary goal of therapy

317

Polling Question

A 45 year- old patient with obesity (BMI 33 kg/m2) and pre-diabetes is having trouble losing weight, in spite of trying to limit caloric intake and incorporating regular exercise. The past history includes a history of depression, seizure disorder and recurrent kidney stones. Current medications include levetiracetam to control seizures and sertraline 200 mg daily for depression.

The best medication choice for weight loss in this patient would be:

• Belviq ® (lorcaserin) 20 mg XL daily • Qsymia® (phentermine-topiramate) starting at 3.75/23 mg and titrating to a maximum dose of 15/92 mg • Saxenda® (liraglutide) starting at 0.6 mg sc and titrating the dose to 3.0 mg daily • Contrave® (bupropion-naltrexone) starting at 90/8 mg, and titrating dosage to a maximum of 2 tablets twice daily

318 Polling Question

A 39 year-old female with a BMI of 35 kg/m2 gives a history of recurrent episodes of out of control eating that have occurred at least once a week since her mother’s death two years ago. Not only has she gained 50 lbs during this time, but she also experiences enormous guilt and hides her eating from her friends and family. She is desperately seeking treatment to control this behavior. You diagnose the patient with binge eating disorder and refer her to a psychologist.

Which medication is FDA approved for treating this condition?

• Qsymia® (phentermine-topiramate) • Belviq® (loraserin) • Saxenda® (liraglutide) • Vyvanse® (lisdexamfetamine) • Contrave® (bupropion/naltrexone)

321

Further Educational Resources

1. Mechanisms of Anti-obesity Medications: Angela Fitch, MD, FACP, FOMA

2. Optimizing Obesity Treatment Using Medication Combinations: Angela Fitch, MD, FACP, FOMA

3. Update on Pharmacologic Treatment Options: Harold E. Bays, MD, FTOS, FACC, FACE, FNLA, FOMA, Dipl. ABOM 4. Download the Obesity Algorithm at ObesityAlgorithm.org

www.OMAcademy.org

324 Fundamentals of Obesity Treatment:

Case Study

161

NUTRITION CASE STUDY

KEY LEARNING OBJECTIVES

1. Analyze a daily intake food log for nutritional composition and health impact 2. Identify risk for nutritional deficiencies 3. Develop a treatment plan for obesity using the 4 pillars with a focus on nutrition

PATIENT HISTORY AND REVIEW OF SYSTEMS

48 y/o female with history of class 1 obesity, T2D, Crohn’s disease, and dyslipidemia presents for an initial visit for weight loss. Crohn’s disease was diagnosed at age 28. She was treated with repeated courses of steroids and relates a 30 lbs. weight gain during this period. She underwent a partial bowel resection and has been in remission since then. She was diagnosed with diabetes 6 years ago. Initially, sugars were well controlled but she states lately her sugars have been much higher. Fasting glucose is typically around 150mg/dL but lately has been closer to 200. Her last HbA1c 6 months ago was 8.8%. She concerned that she continues to gain weight. She recently moved to the area and she said her previous PCP was always on her about her weight. She has tried several commercial programs and lost weight, only to gain it back again. Struggles to avoid snacks at work. Her sugar quickly improved each time she loses weight, if fact she relates that she has had episodes of low blood sugars the last time she tried to diet.

Diet (24-hour recall):

Breakfast Calories Carbs Protein Fat Sugar Fiber Coffee 3 sugars + 3 ½ & 90 2 1.5 5 12 0 ½ 1 sausage biscuits 460 36 11 30 3 1 Orange juice (16oz) 190 39 3 0 39 0 Total 740 77 15.5 35 54 1 Lunch Calories Carbs Protein Fat Sugar Fiber Takeout Kale Salad 251 12 4 22 6 3 Caesar dressing 163 1 1 17 1 0 Diet soda 0 0 0 0 0 0 Total 414 13 5 39 7 3 Dinner Calories Carbs Protein Fat Sugar Fiber Spaghetti Marinara 542 98 19 9 12 9 Meatballs 300 19 13 18 6 3 Italian bread – 2 slices 80 16 2 1 0.5 0.5 Diet soda 0 0 0 0 0 0 Total 922 133 34 28 18.5 12.5 Snacks Calories Carbs Protein Fat Sugar Fiber Donut - glazed 269 31 4 15 15 1 Potato chips 149 15 2 10 0 1 All natural “snacking 210 25 3 11 14 1 chocolate” wafers Total 628 71 9 36 29 2

NUTRITION CASE STUDY

24 hour total Calories Carbs Protein Fat Sugar Fiber Total 2,704 294 63.5 138 108.5 18.5 % Calories 43% 9% 46% 16% Physical activity – stands a lot at work, belongs to a gym but only goes once or twice a month

Sleep – 8 hours

PMH – T2D x 6 yrs., Crohn’s disease – not active,

PSH – Ileocecal bowel resection 23 years ago

Meds – metformin 500 mg ER 2 bid, glimepiride 4gm daily, atorvastatin 20mg daily

All – NKDA

SH – neg Tob/Drug use. Drinks 2 glasses of wine on weekend evenings. Married, 2 children 12 & 14. Occupation – Retail sales

FH – Mother died 74 T2D, HTN. Father 77 HTN, hyperlipidemia, and CAD. Sister 50 T2D, obesity.

ROS – (+) fatigue, dry mouth – tongue feels irritated, numbness in hands and feet

Vital signs – Height 65 inches, Weight 204 lbs., BMI 33.9 kg/m2, Waist Circumference 39 inches,

Neck circumference 16.5 inches. BP 138/92 HR 88 RR 12 O2 Sat 96%

Calculated metabolic activity – REE 1621 kcal/d, Sedentary/limited activity 334 kcal/d, TEF 270 kcal/d, TEE 2215 kcal/d

List primary areas of concern regarding the patient’s current dietary habits. How do the individual components of the patient’s diet compare to acceptable macronutrient distribution ranges?

Areas of Concern:

Diet comparisons:

AMDR Patient % carbs: 45-65% % protein 10-35% % fat: 25-35%

NUTRITION CASE STUDY

What recommendations would you make regarding the specific components of the patient’s diet?

Total calories:

Total carbs:

Total protein:

Total fat:

Sugar intake:

Snacks:

What nutritional deficiency is this patient at risk for and why?

NUTRITION CASE STUDY

The patient expresses an interest in intermittent fasting where she would do 24-hour fast 2 days per week. How would you advise this patient? What medical concerns might you express?

The patient had fasting laboratory testing done before her visit and her results are as follows:

Glucose 188 mg/dL ALT 62 IU/L (0-44) HbA1c 9.8% AST 36 IU/L (0-40) Tot. Cholesterol 234 mg/dL TSH 1.4 HDL Cholesterol 42 mg/dL Vitamin B12 – 144 pg/mL (232-1245) Triglycerides 140 mg/dL Vitamin D – 18 ng/mL (30-100) LDL Cholesterol 164 mg/dL Microalbumin – 22 mg/g creatinine (0-30)

Provide an assessment of this patient’s laboratory results:

Glucose –

HbA1c –

HDL cholesterol –

LDL cholesterol –

ALT/AST –

TSH –

Vitamin B12 –

Vitamin D, 25-Hydroxy –

Microalbumin –

NUTRITION CASE STUDY

Using a patient-centered approach, you both agree on a plan to follow a calorie-restricted diet that is high in protein and limits carbohydrates to under 100 grams per day using a combination of meal replacements, lean protein and increased vegetable consumption. Write down some meal suggestions for each meal.

Breakfast –

Lunch –

Dinner –

Snacks –

Diagnosis

1.

2.

3.

4.

5.

6.

NUTRITION CASE STUDY

Treatment Plan

Nutrition Plan

Physical Activity

Behavior

NUTRITION CASE STUDY

Pharmacotherapy

FOLLOW-UP VISIT #1 – 1 MONTH

The patient returns for a follow-up visit. Her weight is down 12 lbs. and her fasting blood sugar has been in 100 – 120 range off glimepiride. She is only going to the gym once a week but is wearing a wrist activity tracker and has been averaging 8,000 steps per day.

She has tried several different meal replacement bars and comes to the office with 3 different meal replacements she is likes so far.

Item 1 Item 2 Item 3

Sugar alcohol is Maltitol

NUTRITION CASE STUDY

Net Carbs

Total carbs – fiber – sugar alcohol

Compare the 3 nutrition labels for calories, fat, protein and carbohydrate content. Calculate net carbs with attention to sugar alcohol content as well.

Item 1 Item 2 Item 3

Calorie comparison

Fat content

Carbohydrate content

Sugar content

Fiber content

Sugar alcohols

Net carbs

Protein content

NUTRITION CASE STUDY

How would you advise the patient?

Vital signs - Height 65 inches, Weight 194 lbs., BMI 32.0 kg/m2, Waist Circumference 38 inches, Neck circumference 16.25 inches. BP 126/84 HR 82 RR 12 O2 Sat 96% Physical exam - unchanged

Diagnosis

1.

2.

3.

4.

5.

6.

Treatment Plan

General:

Nutrition:

Physical Activity:

Behavior:

Medications:

PHYSICAL ACTIVITY CASE STUDY

KEY LEARNING OBJECTIVES 1. Assess patient physical activity status 2. Utilize the 5 A’s approach for physical activity treatment 3. Develop a treatment plan for obesity using the 4 pillars with a focus on physical activity 4. Write an exercise prescription using the FITTE model

PATIENT HISTORY AND REVIEW OF SYSTEMS

38 y/o male with class 3 obesity, hypertension, T2D, hyperlipidemia and obstructive sleep apnea on CPAP presents has been seeing one of your associates in the practice who referred him to you for obesity management. He has gained 80 lbs. in the 7 years since getting a job promotion which required a lot more business meetings. He played football in high school and college but stopped after his sophomore year due to knee injury. He is interested in bariatric surgery but requires at least 6 months of medical intervention before surgery. He has tried “everything” to lose weight. He has difficulty sticking with a low carb diet. He always feels hungry when he restricts calories. He would like to eat a “healthy diet”. He was in great shape when he played football. He exercised regularly once he recovered from his knee injury but gradually reduced activity as job demands increased. He wants to increase his physical activity but tires very easily. He recently underwent a stress test which was normal. He has frequent lunch and dinner business meetings as well as frequent traveling.

Diet - Breakfast – skipped. Lunch – cheeseburger, fries and diet soda. Dinner – Strip steak, creamed spinach and roasted potatoes. 2 glasses of red wine. Cheesecake for dessert. Beverages – Coffee with 3 packets of sucralose, 2 x 20 oz. diet sodas per day

Physical activity – sedentary at work, no exercise

Sleep – 7 hours when using CPAP but he has not been using every night

PMH – HTN x 5 years, T2D x 4 years, OSA x 5 years, hyperlipidemia x 5 years

PSH – Arthroscopy R knee age 20

Meds – Insulin glargine 30 units daily, nateglinide 120mg ac TID, losartan HCT 100/25 mg daily, metoprolol 100mg daily, atorvastatin 40mg daily, meloxicam 15 mg daily

All – Penicillin - rash

SH – Tob 2 ppd/1-2 glasses of wine daily/denies drug use. Married, no children. Occupation – National sales account executive

FH – Mother 63 alive with obesity, T2D, HTN. Father 65 alive, HTN, Hyperlipidemia, T2D and CAD. Sister 41 HTN.

ROS – (+) fatigue, mild dyspnea with exertion, right knee pain, migraine headaches

Height 75 inches, Weight 355 lbs., BMI 44.4 kg/m2, Waist Circumference 53 inches, Neck circumference 21 inches

PHYSICAL ACTIVITY CASE STUDY

Write down your assessment of his physical activity history. What are some of the positives and some of the concerns you have regarding his activity history?

Positives –

Negatives –

Using the 5 A’s approach, write some examples of how you would begin the discussion on physical activity? How would you assess his current activity?

Ask:

Assess:

How would you advise this patient on ways to increase his physical activity? Using the 2018 Physical Activity Guidelines, what are his recommended activity targets for weight loss?

Advise:

The 4th “A” is Agree. Agree on realistic, well defined goals. In a patient encounter, this would involve a collaborative effort where the patient establishes their goals under your guidance. Write a sample exercise prescription for reasonable exercise recommendations for this patient using the FITTE format.

F –

I –

T –

T –

E –

PHYSICAL ACTIVITY CASE STUDY

The 5th “A” is Assist/Arrange. What recommendations would you make for this patient?

Assist –

Arrange –

List which laboratory tests you would recommend and why.

Diagnosis

1.

2.

3.

4.

5.

6.

Treatment Plan

General

PHYSICAL ACTIVITY CASE STUDY

Nutrition Plan

Physical Activity

Behavior

Pharmacotherapy

PHYSICAL ACTIVITY CASE STUDY

FOLLOW-UP VISIT #1 – 1 MONTH

The patient started walking but is having increased pain in his knee limiting any progress. He has started lifting weight 3 days per week. He has not seen a physical therapist. His weight is down 14 lbs. He has been following the dietary recommendations and is taking metformin ER 500 mg 3 daily. His basal insulin is down to 16 units daily and fasting glucose has been around 110 mg/dL

Height 75 inches, Weight 341 lbs., BMI 42.6 kg/m2, Waist Circumference 51.5 inches, Neck circumference 20.5 inches

Remaining physical exam unchanged

Laboratory results

Glucose 124 mg/dL ALT 92 IU/L (0-44) HbA1c 8.4% AST 55 IU/L (0-40) Tot. Cholesterol 187 mg/dL TSH 2.3 HDL Cholesterol 28 mg/dL Vitamin B12 – 459 pg/mL (232-1245) Triglycerides 268 mg/dL Vitamin D – 21 ng/mL (30-100) LDL Cholesterol 105 mg/dL

Treatment Plan

General:

Nutrition:

Physical Activity:

Behavior:

Medications:

PHYSICAL ACTIVITY CASE STUDY

FOLLOW-UP VISIT #2 – 2 MONTHS

The patient saw a physical therapist. He has started an exercise program to improve leg strength and start using a recumbent bike at a gym. His knee pain has improved. He lost 19 lbs since his last visit, his insulin dose is down to 8 units daily and he stopped nateglinide. His fasting blood sugars are around 100 mg/dL. He has continued on his dietary program.

He complains that his weight loss has plateaued over the last 2 weeks. He has been engaging in vigorous exercise at a gym twice a week for 45 mins.

How does his current activity compare to the 2018 Physical Activity Guidelines?

What are your recommendations?

General:

Nutrition:

Physical Activity:

Behavior:

Medications:

PHYSICAL ACTIVITY CASE STUDY

Over the next 3 months he loses an additional 18 lbs. and the patient elects to undergo a Roux-en-Y gastric bypass. He subsequently loses 100 lbs. over the next 12 months. He has steadily increased his physical activity and is currently doing 60 minutes of moderate intensity exercise 4 days per week and is lifting weights 2 days per week.

How does his current activity compare to the 2018 Physical Activity Guidelines recommended targets for weight maintenance?

In a subsequent follow-up in your office, he has recently gained about 20 lbs since his last visit. While jogging, he tripped on an uneven sidewalk and reinjured his right knee. He was diagnosed with an ACL sprain and he is unable to run and can only walk short distances.

What activity recommendations would you suggest?

BEHAVIOR CASE STUDY

KEY LEARNING OBJECTIVES 1. Utilize the 5 A’s approach for behavioral treatment 2. Apply motivational interviewing techniques using open-ended questions, affirmations and reflections 3. Identify change talk, sustain talk and the “righting reflex” 4. Discuss components of cognitive behavioral therapy and apply to a patient scenario 5. Identify medications that cause weight gain and their alternatives

PATIENT HISTORY AND REVIEW OF SYSTEMS

42 y/o male with class 2 obesity, hypertension, T2D and depression presents for what should be a routine 3-month visit, but his last visit was 6 months ago. His ex-wife told him to follow-up with his physician. That he was fat and lazy and that he needed to start taking care of himself. He mostly feels fine but relates that some mornings he feels shaky, lightheaded and occasionally breaks out in a sweat. He saw your partner last visit who told him he was obese and that he wanted him to lose 10 lbs. before his next visit. He has gained 4 lbs. since his last visit. Looking at his weight graph, he has gained 40 lbs. in the past 5 years. He denies any current depression symptoms. He states depression was primarily due to issues with job change and divorce, though he states he is a little frustrated because was recently passed up for a promotion.

Diet – Skips breakfast, chips and a soda for lunch, meat and potatoes dinner or fast food. Snack in the evening. Usually wakes up during the night and makes a sandwich.

Beverages – Coffee with 3 packets of sugar, 3 x 20 oz. sodas per day

Physical activity – sedentary at work, states he does not have any time to exercise

Sleep – 5 hours interrupted with snack

PMH – HTN x 8 yrs., T2D x 5 yrs., depression during his divorce 4 yrs. ago,

PSH – none

Meds – metformin 500 mg daily, glimepiride 4 mg daily, lisinopril 20 mg daily, paroxetine 40mg daily

All – NKDA

SH – neg Tob/EtOH/Drug use. Divorced x 4 years, lives alone. Has 2 kids 12 & 14. Occupation – Assistant facilities manager, promoted from grounds keeper 5 years ago.

FH – Mother 66 alive with obesity and MI at 59. Father 67 alive with obesity, T2D and CAD. Brother 40 exercises regularly and is in good health.

ROS – (+) fatigue, mild dyspnea with exertion, right knee pain, migraine headaches

Height 70 inches, Weight 265 lbs., BMI 38 kg/m2, Waist Circumference 44 inches, Neck circumference 19 inches

BEHAVIOR CASE STUDY

Using a motivational interviewing style, list some ways in which you would initiate the discussion regarding his weight and health.

Where do you see examples of the patient experiencing stigma and bias regarding his weight?

Using the 5 A’s model, how would you “Assess” his health status?

Reviewing the patient’s dietary history, list some concerns you might have regarding his eating behaviors, and comment on whether or not this patient may have an eating disorder.

BEHAVIOR CASE STUDY

You ask the patient about his interest in losing weight, and he replies: “I know losing weight would help my diabetes, but I don’t have time to make better meals.”

What part of that statement is considered change talk, what part is sustain talk? List some examples of the “righting reflex” health care professionals should avoid.

List some affirmations and reflections to respond to the patient’s statement. Affirmations and reflections often elicit further change talk from a patient and can progress to goal setting. If it does not, what questions might you ask to build on change talk?

You ask the patient if he is interested in losing weight. He replies, “I have been thinking about it. They are starting a weight loss program at work soon and I am planning on signing up for it.”

Using the Stages of Change, what stage is he in and why?

BEHAVIOR CASE STUDY

Utilizing the Readiness Scale, you ask the patient, “On a scale of 1 to 10, with 1 being the lowest and 10 being the highest, how ready are you to make changes in your diet”? He replies a “6.” What is the best way to respond and why?

Using the 5 A’s model, list some statements you might make on how you would “Advise” the patient regarding his weight and health.

List some Cognitive Behavioral Therapy recommendations you might make for this patient.

Self-monitoring:

Problem solving:

Stimulus control:

Stress management:

Social support:

Cognitive restructuring:

BEHAVIOR CASE STUDY

How would you address the treatment of the patient’s depression?

Diagnosis

1.

2.

3.

4.

5.

6.

Treatment Plan

Nutrition Plan

BEHAVIOR CASE STUDY

Physical Activity

Behavior

Pharmacotherapy

MEDICATION CASE STUDY

KEY LEARNING OBJECTIVES

1. Identify medications associated with weight gain and their alternatives 2. Develop a treatment plan for obesity using the 4 pillars with a focus on medication 3. Analyze the risks and benefits of anti-obesity medications in a clinical setting PATIENT HISTORY AND REVIEW OF SYSTEMS

A 63-year-old African American female with hypertension, prediabetes, hyperlipidemia, class 3 obesity, depression, and osteoarthritis is referred to you for obesity treatment by her orthopedic surgeon. She has advanced degenerative joint disease of her knees. She is being considered for bilateral knee replacement, but her surgeon would like her BMI below 35 kg/m2 before she has surgery. However, despite her best efforts, she has gained 20 lbs. over the past year. The patient has tried numerous diets in the past with only temporary success. Her activity is very limited due to knee pain and needing a cane or walker for ambulation. She sleeps 6 hours per night, waking up frequently due to knee pain.

Diet – Breakfast – biscuit and gravy. Lunch – Tuna fish sandwich. Dinner – fried fish and tater tots. Beverages – coffee with half & half and sugar (3 packets), 16oz sweet tea 3 times a day. Snack – 3 oatmeal cookies.

PMH – HTN x 25 years, Prediabetes, Hyperlipidemia x 15 years, Osteoarthritis 12 x years, major depression disorder x 10 years currently stable

PSH – Hysterectomy age 44, cholecystectomy, breast biopsy - benign

Meds – Amlodipine 10mg daily, valsartan 80mg daily, metoprolol succinate 100mg daily, HCTZ 25mg daily, rosuvastatin 40mg daily, gabapentin 300mg TID, Celecoxib 200mg daily, fluoxetine 20 mg HS, olanzapine 10mg daily, Tylenol PM 2 HS

All – NKDA

SH – Neg Tob/EtOH/Drug use. Occupation – retired school teacher.

FH – Mother died 81 colon cancer, HTN. Father died 64 lung cancer, HTN, T2D. Sister alive 61 obesity, T2D, breast cancer, Brother alive 57 HTN, OA knees, hyperlipidemia, MI age 55.

ROS – (+) Fatigue with daytime somnolence, neck, shoulder, hip and knee pain, constipation, edema

PE – Height 65”, Weight 245 lbs., BMI 40.8 kg/m2, WC 48”, Neck size 17”,

BP 152/94, HR 73, Temp 98.4 O2 Sat 95%

Gen – Awake and alert with central adiposity

HEENT – Malampati score 4

Nk – hyperpigmentation of skin

MEDICATION CASE STUDY

H – RRR

L – CTA

A – Soft, nontender, RUQ cicatrix

Vasc – pulses intact, +1 edema b/l

MSK – Knees – swelling & crepitus b/l with increased valgus deformity. Tenderness over lateral meniscus b/l.

Neuro – intact

Psych – denies anxiety or depression for years. Her initial episode 10 years ago associated when she was severely depressed after the traumatic loss of a child.

What diagnostic testing would you order and why?

Review each of her medications and discuss potential impact on weight and alternative options if appropriate.

Amlodipine –

Valsartan –

HCTZ –

Metoprolol –

Rosuvastatin –

Gabapentin –

Celecoxib –

Fluoxetine –

Olanzapine –

Tylenol PM 2 HS –

MEDICATION CASE STUDY

Diagnosis

1.

2.

3.

4.

5.

6.

Treatment Plan

General

Nutrition Plan

Physical Activity

MEDICATION CASE STUDY

Behavior

Pharmacotherapy

FOLLOW-UP VISIT #1: 2-WEEKS

The patient returns to review her diagnostic testing results. She has been following your nutritional recommendations. She is complaining of increased hunger since staring her diet and is interested in taking a medication to reduce appetite. She is having some difficulty with sleep off of diphenhydramine. She is scheduled for a sleep study. She also relates strong cravings for sweets yet. Her knee pain is stable. She started an aquatic aerobic program at a local pool. Denies any GI side effects. She has lost 7 lbs. She denies any increase in depression symptoms.

2 PE – Height 65”, Weight 238 lbs, BMI 39.6 kg/m , WC 48”, BP 152/94, HR 73, Temp 98.4 O2 Sat 95%

Remaining exam is unchanged

Laboratory Glucose 133 mg/dL GRF 56 ml/min/m2 (Stage IIIa) HDL cholesterol 48 mg/dL HbA1c 6.8% ALT 38 U / AST 32 U LDL cholesterol 104 mg/dL BUN 42 mg/dL Total cholesterol 176 mg/dL TSH 3.2 Creatinine 1.2 mg/dL Triglycerides 122 mg/dL Vitamin D 18 CBC – WNL

Describe your interpretation of her laboratory results.

MEDICATION CASE STUDY

Consider each of the following medications. What would be the risks and benefits of each medication choice in this patient?

FDA approved

Phentermine (approved for short-term use) –

Liraglutide –

Lorcaserin –

Bupropion/naltrexone –

Phentermine/Topiramate –

Orlistat –

Off-label Medication (Not FDA approved for obesity)

Metformin –

Topiramate –

Naltrexone –

Bupropion –

Zonisamide –

MEDICATION CASE STUDY

Treatment Plan

General:

Nutrition:

Physical Activity:

Behavior:

Medications:

FOLLOW-UP VISIT #2: 6 WEEKS SINCE INITIAL VISIT

Patient’s knee pain has been stable on topiramate. Her depression stable on lower dose of olanzapine. Patient states her hunger is less on the medication. She saw an RD and made additional changes in her diet. Her weight is down 15 lbs. since her first visit. She is tolerating three metformin ER 500mg daily. Sleep is still poor. She had a sleep study and was diagnosed with obstructive sleep apnea. She is in the process of getting set up for CPAP.

2 VS BP 128/84 HR 78 RR 12 T 98.8 O2 sat 94% Wt 230 lbs. BMI 38.3 kg/m

Treatment Plan

General:

Nutrition:

Physical Activity:

Behavior:

Medications:

MEDICATION CASE STUDY

FOLLOW-UP VISIT #3: 10 WEEKS SINCE INITIAL VISIT

She has been following her dietary program and lost an additional 12 lbs. for a 27 lbs. total weight loss. Her BMI is now 36.3 kg/m2. Her depression is stable off olanzapine. She is complaining of occasional diarrhea. Sleep in improved with CPAP.

2 VS BP 128/84 HR 78 RR 12 T 98.8 O2 sat 94% Wt 218 BMI 36.3 kg/m

She is continued on the same treatment plan.

FOLLOW-UP VISIT #4: 14 WEEKS SINCE INITIAL VISIT

The patient hunger is still controlled. She is following the nutritional recommendations. She has lost 8 lbs. since her last visit. She denies depression.

2 VS BP 128/84 HR 78 RR 12 T 98.8 O2 sat 94% Wt 210 BMI 35.0 kg/m

Total weight loss 35 lbs. or 14% of body weight

Is continued use of her anti-obesity medication indicated? Why or why not?

Treatment Plan

General:

Nutrition:

Physical Activity:

Behavior:

Medications: